Dagny Taggart S Ultimate Guide To GMAT Preparation

Published on September 2021 | Categories: Documents | Downloads: 1 | Comments: 0 | Views: 99
of 56
Download PDF   Embed   Report

Comments

Content

 

 

Dagny Taggart’s Ultimate Guide to GMAT Preparation GMAT Basics:

The Graduate Management Admissions Test is the standardized exam used by graduate business schools for admissions decisions. It is designed and produced by the Educational Testing Service (ETS) and administered through the Graduate Management Admissions Council (GMAC). The three-hour, computer-adaptive test is almost exclusively multiple-choice and yields four scores: Total, Verbal, Quantitative and Analytic Writing Assessment. What the GMAT Measures:

GMAC claims the GMAT can measure basic analytical, quantitative and reasoning abilities developed over time. However, according to the GMAC web site, the GMAT does not measure: “every discipline-related skill necessary for academic work, nor does it measure subjective factors important to academic and career success, such as motivation, creativity, and interpersonal skills.” It continues, “GMAT scores cannot be used to estimate potential for success in a career, because many factors other than basic verbal and mathematical abilities influence work performance.” Test Administration:

The GMAT switched to a computer adaptive testing (CAT) format in 1997. Since that time, multiple problems with cheating, computer system failure, and scoring errors have been identified. In 2000 ETS admitted that scores for 3% of all tests taken between February and March were miscalculated, but students were not notified until December. Common pencil-and-paper test-taking strategies cannot be used on computer delivered exams. Test-takers are unable to underline text, scratch out eliminated choices or work out math problems on screen. Studies also indicate that computer screens take longer to read than printed materials, and that it is more difficult to detect errors on computer screens. Taking the GMAT costs applicants $200 each but colleges pay nothing to receive scores. Since 1985, GMAT has increased fees by $170 per test but offers no financial assistance for test takers of limited financial means. GMAT Grading and Scoring:

The Verbal and Quantitative sections are each scored on a scale of 0- 60. A Total score is calculated by adding Verbal and Quantitative scores then converting this number to a 200-800 point scale. The Analytical Writing Assessment (AWA) is scored on a 1-6 scale. This section, in which students have an opportunity to display creative, innovative and concise writing skills, is graded by two “evaluators,” one of which is a computer. . Predictive Validity and Margin of Error:

GMAC has been able to validate the GMAT for just one purpose: predicting first-year graduate school grades. But GMAC concedes that the GMAT can predict less than 17% of the variation in these grades on average. Independent researchers put the percentage much lower at between 4% and 9%, meaning that over 90% of what determines how well a student will do in their first year in business school is not captured by the test. According to GMAC’s own studies, all graduate schools have access to a better predictor of success undergraduate GPA’s. GMAC’s validity studies use all four scores (Total, Verbal, Quantitative and Analytic Writing) to assess predictive validity. Many schools, however, do not consider Analytic Writing scores in making admissions and financial aid decisions, thereby lowering the test’s minimal added value. An ETS report concludes, “including the Analytical Writing score as part of an admissions screening battery would substantially increase the number of eligible women” because females score higher than males on this section of the GMAT. According to GMAC, two students’ scores have to differ by at least 41 points before they indicate a difference in the abilities measured by the GMAT which means that colleges cannot determine the better qualified of two students whose scores are 530 and 570. Admissions Cutoff Scores:

Due to the GMAT’s weak validity and lack of precision, the GMAC Code of Ethics states: Avoid the use of cutoff scores. scores . Cutoff scores should only be used when there is clear empirical evidence that a large proportion of the applicants scoring below the cutoff scores have substantial difficulty doing satisfactory graduate work. GMAC continues, “In addition, it is incumbent upon the school to demonstrate that the use of cutoff scores does not result in the systematic exclusion of members of either sex, any age or ethnic groups, or any other relevant groups in the face of other evidence that would indicate their competence or predict their success.” In addition, the National Association of Graduate Admissions Professionals’ handbook Professional Standards and Practices states, “It is advisable to consider implementation of additional practices that eliminate discrimination, provide equal access and treat tests as a

This website stores data such as single component of the admissions procedure since test scores are not the sole measure of potential success.” Despite this strong guidance, cookies to enable essential site hundreds of schools use absolute cutoff scores for admission and financial aid. For example, Texas Christian University has an accelerated MBA program that requires a 620 on the GMAT. Chapman College of Business and Concordia functionality, as well as marketing, University each demand a 500, while California State University at Stanlihaus mandates a 450. The University of North Florida is even personalization, and You more explicit: “aanalytics. minimum 20 verbal and 22 quantitative GMAT sub-score is required for graduate admission.” GMAC recognizes theseatabuses are widespread, noting among “Business School Application Tips” in one publication, “Minimum Score may change your settings any time Requirements: Some schools require minimum scores on the Graduate Management Admission GMAT. Other schools only have or accept the default settings. minimum requirements for certain sections of the GMAT. Know the minimum requirement at a particular school, if one exists, so you

only apply to schools you are qualified to attend.” GMAC takes no action against schools that misuse its test results. In fact, schools with cutoff scores are listed in GMAC’s college search and financial aid identification programs and given all of the benefits of GMAC membership.

Privacy Policy

GMAT ‘Merit’ Scholarships:

Marketing Millions of dollars in “merit” scholarships are based on GMAT cutoff scores. For example, the University of Oregon’s Lundquist College of Business Scholarship requires a 650 on the GMAT. Many aid programs consider only the GMAT “Total” score (Verbal + Quantitative), potentially decreasing the number of women by excluding Analytic Writing. Personalization The average cost to attend business school is a little over $10,000 per year. Top 25 schools however, cost more than double that, Analytics

potentially out many of aid thebut 95% of Black 80+% of Latino who report they need financial aid. In contrast, Whites are pricing least likely to need most likelyand to get it from GMAT test-takers score-based “merit” scholarships. Business School Rankings and the GMAT: Save Accept All

http://www.totalgadha.com

 

  Rankings publicized by magazines such as U. S. News & World Report and Business Week contribute to the illusion that GMAT scores are a valid measure of the worth of graduate management programs and their students. The average GMAT scores of enrolled students count for almost one-sixth of a school’s total weight in both publications. Nearly a quarter of all MBA applicants say these rankings are  “extremely important” for deciding where to apply. Many business schools point to rising sc scores ores as evidence of higher standards, neglecting to put them in the context of the rapid climb of average GMAT scores — 57 points in the past 18 years. Women and the MBA:

Women graduate from college with higher undergraduate Grade Point Averages (GPAs) than men but score 38 points lower on the GMAT- - a gap that has grown by 28 points since 1982. Women who graduate from Business School have, on average, GPA’s equivalent to those of men. Independent research indicates that the GMAT over-predicts the performance of men and under-predicts that of women. Test scores play a role in school choice. Men are much more likely than women to send their GMAT reports to “Top 20” schools and define such a school as their top preference. Women submit fewer applications and apply to less selective schools than their male counterparts. The number of women in business schools peaked at 30% and remained flat for several years. GMAC reports that while 87% of MBA programs received more total applications for 2001-2002, less than half of these schools saw an increase in applications from women. Almost one-third of business schools at private universities have seen a drop in female applicants. African Americans and Latinos:

In 2000-2001 African Americans scored, on average, a verage, 111 points lower than Whites. Latino populations scored between 64 and 87 points lower than Whites. The huge score gap between Blacks and Whites has narrowed only slightly in two decades, while the difference between Latinos and Whites has widened. In 1994-1995 on ly 143 Blacks scored higher than 650 (over 100 points lower than the averages for top-tier schools). High-scoring Whites outnumbering high- scoring Blacks 75 to 1. Without affirmative action, African Americans and Hispanics would be much less likely to get into a top-25 graduate business school. Members of these groups enter the MBA pipeline with far more concerns regarding their abilities to meet academic and curriculum demands, fears reinforced by much lower GMAT scores. African Americans and Hispanics are greatly disadvantaged by a test with little predictive ability and multiple opportunities for abuse. Other Groups:

Applicants who speak English as a second language (ESL) are disadvantaged by the primarily multiple-choice nature of the GMAT and the fast pace at which items must be answered. ETS studies show that when these characteristics are not part of the test (as in the Analytic Writing section) the score gap between whites and ESL applicants is greatly reduced. GMAC admits that average scores decline for test-takers over the age of 31. There is also an inverse relationship rela tionship between GMAT scores and business experience. Studies conducted for GMAC show GMAT scores under-predict the performance of older students (particularly women) but demonstrate that business experience directly contributes to success in MBA programs. AVERAGE GMAT TOTAL SCORES 2000 – 2001*

*most recent year data is available All Test-takers 527 Women 503 Men 541 Whites 538 Blacks 427 Latinos (several nationality groups) 451-474 While little research has been done on the validity of GMAT scores for students with disabilities, the test is likely to have the same discriminatory impact and low predictive validity as similar standardized admission tests produced by ETS such as the SAT. Alternatives:

GMAC and ETS have long been the national gatekeepers for business school admissions. Soon there might be some competition. Researchers at the University of Michigan’s School of Business have developed alternative measures of managerial potential that they claim better predict success in both academic and practical endeavours. This new assessment approach, dubbed the “Rainbow Project,” also claims to produce far less racial and gender bias than the GMAT. In 1985 Harvard Business School (HBS) decided to eliminate the GMAT from their admissions process. John Lynch, the Admissions Director at the time, gave several compelling reasons. In a blind test, Harvard found that admissions decisions made with and without

This website stores such the as same. Success at Harvard depended on intangibles such as motivation, interpersonal skills, the GMAT were data essentially perseverance and hard work cookies to enable essential site - - all factors not measured by GMAT. Looking at undergraduate grade-point average (UGPA), ethics, leadership, community activities, functionality, as well as marketing, prior work experience and the interview made GMAT scores “superfluous”. Harvard was also concerned about the perceived emphasis applicants place on the GMAT and that strong applicants with scores below the 99th percentile personalization, and analytics. You Lynch also pointed out that an “artificial barrier to the admission of qualified but poorer students is were intimidated from applying. unacceptable.” may change your settings at any time Eleven later Harvard reinstated the GMAT, saying the decision “reflects the fact that there have been significant improvements to or accept theyears default settings. the structure of the test”. The only changes made since 1954 have been the addition of the AWA, which is used only sporadically and the move to the Computer Adaptive Test, which came with multiple problems. Harvard also pledged to work closely with GMAC to cooperate “on ways to further change the test” to focus on intangible qualities. In six years, no further mention has been made of Privacychanges Policyto the test. Guide to Making Changes

Marketing Schools that have de-emphasized standardized tests in admissions have done so for a wide range of reasons and have chosen several different approaches. The following steps serve as a guide: Personalization 1. Conduct a standardized test “audit” to understand how your school uses the GMAT and clarify current test uses for admissions and scholarship awards. Evaluate these goals in relationship to the overall mission of the school. Consider Analytics

broader admissions issues, including diversity, outreach, and cost. 2. Conduct an independent validity study and compare your access experience with the GMAT with the validation va lidation and correlation studies conducted by GMAC. GMAT scores provide useful information only to the extent that they lead admission officers to make differentand betterSave Accept All decisions than they would make without the test.

http://www.totalgadha.com

 

  3. Assess the impact of a variety of admissions requirements on different applicant groups such as women,Latinos, African Americans, older applicants and students with disabilities. 4. Evaluate the predictive value of alternate criteria that influence college persistence and success such as UGPA, work experience and motivation. 5. Evaluate the impact of GMAT requirements and/or minimums on self-selection, se lf-efficacy and school choiceparticularly on underrepresented groups. 6. Develop admissions alternatives, including: make submission of GMAT scores optional; require the GMAT only for those applicants with UGPAs below a certain point; and drop the GMAT completely. Hard Facts About Your GMAT Score

See how the chart below shows GMAT scores in 3 Tiers.

This website stores data such as cookies to enable essential site functionality, as well as marketing, personalization, and analytics. You may change your settings at any time or accept the default settings.

PrivacyHere's Policy a follow-up to the last chart. Again 3 tiers of applicants regarding GMAT scores. Marketing Personalization Analytics Save

Accept All

http://www.totalgadha.com

http://www.totalgadha.com

 

 

Here's an interesting graph from Kellogg's 2004 entering class. This is a good GMAT question possibility. If 5282 applied to the 2-year MBA class resulting in 469, what percentage of students were admitted with the various GMAT brackets of scores as shown in the graph? When you do the math (Taking into account that Kellogg admits 12% more than needed for instances where the applicant chooses another school), you get the following: 640 or less GMAT = 1056 apps with 42 accepted at 4% acceptance. 650-690 GMAT = 1584 apps with 158 accepted at 10% acceptance. 700-740 GMAT = 2059 apps with 252 accepted at 12.2% acceptance. 750-780 GMAT = 581 apps with 74 accepted at 12. 7% acceptance. That means the 2004 class cla ss had 25% of all students who scored 700 or more accepted. However, the actual student population is actually 62% people who scored 700 or more. This is actually a great challenge and encouragement as I thought it was much worse. So 38% of the class scored 690 or less? That's cool. 700-740 is only 2% more likely to receive acceptance than those who had 650-690. Not much of a difference. I think this is why we ought not to kill ou rselves for not getting 700 and for not killing our self if we do get 700 or more and don't get into a top Business School. That 2% can't hurt though.

The graphic below is part of how student needs are assessed. Each red zone is a plateau. It is a score range that is REALLY hard to break through. You are in the plateau that the course is designed to get people to. Don't believe the nonsense out there. There is no super effective one-size-fits-all solution to maximum GMAT improvement. Achieving and moving through each plateau has its own unique set of needs and concerns.

Study tips

1.  Rec ord all mistakes on wrong questions and revisit them no sooner than 5 days. If still getting wrong, then zero on the subset's and go for more help.

2.  Always read the explanation in the book to see why you get it wrong and take enough time to really understand the concept. This website stores data such as 3.   Pick essential a date and site register. Don't study then register. There was a tendency to not take prep time as seriously. cookies to enable 4.  as Quite fewmarketing, went through OG questions more than once. Not all questions on the repeat though; though; Mainly verbal sections and functionality, wellaas personalization, andthat analytics. Youof weakness. Quant were areas may change your settings at any 5.  Wrote practice essaystime at least 20 times under timed conditions. or accept the default settings. 6.  Visit the test center before the day of test. Many did so a week or two before. 7.  Time spent by most averaged between 3 and 4 months. 8.  All averaged 2-3 hrs per day and no less than 10-12 hrs on the weekends.

Privacy Policy

9.  Took advantage of every break during during the GMAT. Take a bathroom break, some washed their face to wake up up,, got fresh air.

10.  Study in blocks. One person mentioned studying in 80 minute blocks without getting up to to build stamina. Marketing

Another person

mentioned studying in 75 minute blocks with exact numbers of questions that are on the real GMAT to build stamina and skill.

Personalization

Many agree that the ability to handle the time constraints is nearly as important is solving the problem. Scores in the actual

Analytics

GMAT are worse if you leave an answer blank than if you get the answer wrong.

Save

Accept All

http://www.totalgadha.com

 

  Interesting note: The Official Guide for GMAT contains questions that are in the 550 to 650 difficulty range.

Let’s Get Started 

Use a track sheet that will help you to zero in on on the what you can target for improvement. improvement. Instead of saying, I need to improve in math you’ll be able to say, “I need more work in probability, more work in stamina or more work in seeing various forms of misplaced modifiers in sentence correction questions. One thing is for sure, most people find that 50% or more of the reason why they get a question wrong is due to carelessness, or a loss/lack of stamina. Without a good tracking system, you may no nott be able to measurably know by how much this is true or whether it’s it’s something else. It’s also good to make the notes area fr frequently equently to note the kind of subcategories of question types or if it was a concept you didn’t know or a concept you did know, but weren’t solid on yet. When you start out, it’s helpful to spend spend a week or two un-timed. Then as you progress, start putting y yourself ourself under moderate time pressure to at least record how long it’s taking you to do a particular question. It would be completely ridiculous to record how long it took you to answer every question. It’s far better to track yourself yourself in blocks of time. For example, with 12 questions, you should have X minutes left and with 24 questions, you should have X minutes left and so on. Another excellent tracking method is to p put ut one, two or three X’s in the slow box if you took longer than real GMAT time conditions to answer the question. Another good strategy is to go through questions in the following manner. 1)  Do about 40 questions at a time. In this way, you sho should uld be giving yo yourself urself 80 or so minutes to complete them. them. This will help you to get used to doing questions in blocks similar to the GMAT. This will build your physical stamina and build longer periods of mental toughness and focus. 2)  Check your answers and DON’T look at the explanation of the answer. Circle the right answer in red and the sheet. 3)  Go back, and use the teaching method as explained explained earlier to resolve the problem. Note whether you find the reason why you got the answer wrong as careless or due to concept error in the chart. A concept error is defined as an error where you didn’t understand the theory enough to know which steps steps and in what order to solve the pro problem. blem. It could also be a relevant piece of knowledge that you forgot forgot or haven’t learned yet. See the sample posting above tto o get an idea of what a teaching method solution looks like. 4)  Next go through the explanations to try and understand if you your methods were right or if the book offers an alternative solution method. There is more than one way to explain or solve many problems, so don’t feel like you have tto o follow the particular explanation in the book. If your method solved it, and yo you u can repeat it, then stay with what you know as long as it it holds up to scrutiny when slight changes in the problem problem don’t allow your method to to work. In which case, you’ll need to adapt the book concept or dig deeper into asking yourself whether you really understood the concept to begin with. By the way. It’s really helpful to ask yourself two questions when you’re done with each test problem re-solve 1) would I be willing to bet $20 that I really understood the concept being being tested? 2) Did I write out my explanation well enough that most anyone could look at it and understand how I solved the problem?

What Resources should I use?  use?  

This website data Guide such to asthe GMAT The Official •  stores cookies to •enable essential   Download 1000site RC,CR,SC and PS files from www.totalgadha.com www.totalgadha.com   functionality, as well as marketing, personalization, and analytics. You Should I use a Test Prep Company? may change your settings at any time It’s really up to you. If you find that you struggle with discipline or life is really really busy. or accept the default settings.

Then you should.

What kind of schedule should I use? What order should I go through the material?

Again, it’s really up to you, but here is a basic plan that tha t most people have used that showed great results. Privacy Policy www.totalgadha.com FIRST.  FIRST. It’s the best Basic rreview eview of Math and Verbal. 1)  Go through the Total Gadha study lessons www.totalgadha.com

Marketing 2)  Go through the Total Gadha’s www.totalgadha.com www.totalgadha.com Quant  Quant and Verbal Study Guide section 3)  Do at least 50 questions of each type. Personalization 4)  Go through the Official GMAT Guide. Do all the questio questions. ns.

Analytics

5)  Take time to do a weekly review of the verbal and quant formulas and reference sheets you made.

6)  Do at least a dozen practice essays. Do each one under timed conditions.

Save

Accept All

http://www.totalgadha.com

 

  7)  Don’t study the day before before the test. And make sure to visit the test cent center er at least a week before the test. Bring ear plugs to the test center if noise will be an issue. Get used to studying with them if yo you u do though. 8)  Create regular review sessions where you go over all the questions you got wrong on on a weekly basis. Make a binder or something similar.

Number Property Table

Integer

Positive

0

X

1

X

X

2

X

X

7

X

X

-3

X

Negative

Even

Odd

Prime

X X X X X

.5

X

Pi

X

√ 6

X

-19/2

X X

X

X

51/3

X

X

Even negative x Odd negative

X

X

Odd negative + Odd negative

X

Even Positive – Odd Negative

X

Odd negative / Even positive

X

X

X X

X

X X

X

Points of emphasis:

0 is an integer, it is neither positive nor negative. Negative numbers are not prime 0 and 1 are not prime

Plan Your Preparation

There are many materials available. Its important that you choose the right material and not regret on wasting time on some unwanted material. These materials are considered the best. Schedule: 20 Weeks to GMAT

This website stores data such as A proper planessential and commitment cookies to enable site towards implementing is the next step. This schedule is just a sample. You can tailor it as per your convenience and competency level. functionality, as well as marketing, Some Tips : personalization, and analytics. You •  Practice Practice  the material in the form of Timed Practice tests. Track the day, test, answers,accuracy, average time taken for may change your settings at any time each question in an excel sheet. Create a sheet for every test you take. or accept the default Analysis   is thesettings. most important part. Spend as much time as possible in understanding each and every option. Its more •  Analysis important to know why an option is wrong wrong than why an option is right. GMAT OG explains why the other options are wrong. The very same concepts are tested. When you analyze make sure to:   Find why you went wrong. Check if you know tested concept? Check the material on the tested concept. Privacy Policy   Check if your reasoning for every option matches with the given reasoning. series doesn’t have any explanation. You will have to resort to online Forums. Try to find why each option is not •  1000 series  correct. Marketing performance  time to time. Accuracy doesn’t increase over night. It happens with time and a nd practice. Be •  Check your performance patient. It happens not by solving more questions but only when you learn from your mistakes. Personalization Forums

Analytics

The forums play an important role in bringing many people onto the same platform. Some of these members are dedicated and contribute useful information.

Save

Accept All

http://www.totalgadha.com

 

  Efficient usage of forums. Efficient doesn’t mean over posting. posting. Who cares how many ‘A’ or ‘B’ or ‘Agree’ you post? There is no

award for highest posters, but there will be admirers for every good post. You end up wasting time for all those unnecessary posts. Your objective is to learn and not being highest poster. Approach to GMAT SC Search:  Make use of search instead of posting every question as new thread. There will be enough posts on every question. You will Search:

be able to read lots of queries and answers that you may not be able to by posting a thread. Post  explanation  clearly. Try to answers to queries, give full explanation. There could be some misleading information as well. If Post your explanation

you differ from any other explanation, post it. By doing so, you can correct others, correct yourself. Don’t solve every post on forum. forum . Don’t solve them unless you have solved them as part of your schedule earlier. If you solve them

now, you will know answers while solv ing them in timed tests. This will affect in checking your accuracy a and nd progress. Track Ur posts posts:: Track all your posts through Excel. Create 4 columns: Date, Post link, Post type(query,explanation), Status( open,

Resolved) Question Format Process of Elimination

One of the most useful skills you can master for any standardized test is Process of Elimination (POE). For every question on the GMAT, there are five answers, but only one of them is deemed .best. by GMAC. (But ’best’ is rat her subjective, especially when it comes to GMAC and verbal questions.) Often the easiest way to identify the right answer is to find all the wrong answers and eliminate them. POE is very useful in verbal because often the best answer isn’t all that great. While the credited response might suck, there isn’t anything in it that makes it wrong. There’s nothing in it you can point to and say this is wrong .Focus your energy on getting rid of the answers that you know are wrong. We’ll go over the individual problems that wrong answers are most likely to have in the appropriate topics and lessons. And remember, use your erasable note board for POE. Since you can’t cross off the wrong answers on the screen, write ‘ABCDE’ on your note board and cross off choices as you eliminate them One of the keys to successful POE is to avoid creating stuff. A great many of the wrong answers in the verbal section, especially in critical reasoning, are tempting only if you work to connect the answer to the question.As soon as you start working to justify an answer choice, or start telling some story to connect an answer back to the question or the argument, you’re almost certainly dealing with a wrong answer. The right answer doesn’t need any help from you to be right; it is supported by the information in the passage or argument or question Ultimately, even the verbal part of the GMAT is objective. Objective? Well, maybe not exactly. But the fact is that there are definite reasons why every right answer is right and every wrong answer is wrong. The key to success on the verbal section of t he test is figuring out what rules GMAC uses to determine the right and wrong answers.  answers. 

Focus on why GMAC thinks answers are wrong, and the verbal section will w ill become a breeze. Identify Wrong Answers

Sentence Correction questions present you with a sentence that has a portion underlined. Your is to decide if that underlined portion contains a grammatical error. Be careful--just because you speak English well does not mean that you know the rules of grammar. These basic facts will further introduce you to the sentence correction section

job

• The underlined portion of a sentence may or may not contain an error. • The first answer choice always repeats the underlined portion exactly. • The other answer choices represent potential corrections to the initial sentence.

• Choose the first answer This website stores data suchchoice as only when you can find no error in the original sentence and can find errors in the cookies other to enable essential site four answer choices. • More than one as answer choice may correct an error that exists in the given functionality, as well marketing, sentence. personalization, and analytics. You • Some answer choices correct the initial error but introduce a secondary grammatical error--eliminate these answers. may change settings any time • Some your answer choicesat correct the initial error but re-word the sentence so that the meaning is changed--eliminate these. • Eliminate wrongsettings. answers and choose the choice that is left. or accept the default Questions  The Basic Approach to Sentence Correction Questions 

the sentence and look at the answer choices. Privacy• Read Policy

• Does the sentence contain a grammatical error? Look for the frequently tested errors you’ve learned. • Eliminate answer choices based on your evaluation of the 2/3 split and any answers that contain grammatical errors. Marketing • Evaluate the answer choices you have left. Look for differences and determine which ones are wrong. • Don’t fall into these traps on Sentence Correction questions! •Personalization Don’t pick the answer choice that sounds right! It may contain grammar mistakes. • Don’t think that you must find an error. Answer choice (A) can be the credited answer.

Analytics

• so about caughtthe up little in looking error in the question that you forgetthey’re to use often the answer choices to you’re help you. • Don’t Don’t get forget errorsfor likethe passive construction and subjunctive; important when down to two. •Save Do stay calm andAccept focused. All Find the four answer choices with errors and eliminate them.

you

http://www.totalgadha.com

 

  Some most commonly observed similarities in the questions. Remember they WILL not be same in every case.

• Answer choices in which the word ‘being’ is a verb are rarely correct. • ‘There’ constructions are rarely correct. non-countable things and answer: How much?. [soup] •   ‘Less’ and ‘amount’ refer to non-countable • ‘Fewer’ and ‘number’ refer to countable things and answer: How many?. [people] • ‘Like’ or ‘unlike’ almost always trigger a comparison trap. Make sure that apples are being compared with apples • All comparisons require parallel structure (analogies, metaphors, similes) • If you have to guess, go for the shortest answer that is clear and unambiguous. • Sometimes the best way to correct ambiguous pronouns is to avoid pronouns altogether. • A descriptive phrase at the beginning of a sentence set off by a comma is a trap; what follows the comma should be the subject of the phrase. • A descriptive phrase at the beginning of a sentence set off by a comma [usually signals] a [modification] trap; what follows the comma should be the subject of the phrase. An Approach to Solve Sentence correction

Look for a 2/3 Split in the answer choices.  Most of the time, two of the answers will try to correct the sentence in one way  way  and the other three will try to correct the sentence in another way. This ’2/3 Split’ is v ery useful in helping you determine what grammatical issue is being tested. •  Using the 2/3 Split and your assessment of the question, eliminate answers that fail to correct the error you found. •  Evaluate the remaining answer choices. •  Eliminate those that introduce secondary errors. •  Examine all the differences among the remaining answers and choose the best one. Here C is best

Look at the 2/3 split. Should we go with ’they’ or ’the codes’. They: Are you sure you know what ’they’ refers to?. The Codes : We must use ’the codes’ to eliminate the ambiguity with the word ’they. Which is the best answer between D and E? Look for secondary errors D : No. This answer changes the meaning of the original sentence by adding the language ’able to be’. E : Yes. This answer corrects the initial error and introduces no secondary mistakes. Just because a sentence ’sounds right’ does not mean that it is grammatically correct. Our ears are trained by everyday speech, and everyday speech is frequently grammatically wrong. GMAC knows many cases of sentences that sound right but are actually wrong, and they will utilize these special cases to trap you. Instead of using your ear, always try to apply a rule of grammar. Don’t forget that a sentence can be correct as written. Sometimes GMAC will use a sentence that ’doesn’t sound right’ but is actually correct. Notice that the wording of the sentence is somewhat stilted and formal. But can you find a grammatical error? When you can’t find an error, you should look at the 2/3 split for clues as to what issue might be being tested. In this case, which is correct: ’is’ or or  ’are’?

This website stores data such as In this sentence, ’is’ is the correct verb because the word ’either’ followed by cookies to enable essential site singular choices takes a singular verb. So the correct answer has to be either (A) or (B). Canas you findas a secondary functionality, well marketing,error in one of them? ’as being’ in answer (B) is incorrect, soand (A) analytics. is the correct answer. personalization, You may change your settings at any time Basic English  English   or accept the default settings. Parts of a sentence description Adjective - Describes things or people.

PrivacyAdverb Policy- Alters the meaning of the verb slightly Articles- a, an (indefinite articles)

the (definite articles) Marketing

Conjunction- Joins words or sentences together Interjection-A short word showing emotion or feeling Personalization Noun- names things Preposition- Relates one thing to another

Analytics

Pronoun- Used instead of a noun to avoid repetition

Proper noun (subject). The actual names of people or places etc.

Verb- Action or doing word

Save

Accept All

http://www.totalgadha.com

 

  Subject

The subject is the person or thing the sentence is ’about’. Often (but not always) it will be the first part of the sentence. The subject will usually be a noun phrase (a noun and the words, such as adjectives, that modify it) followed by a verb. e.g.: David works hard. Who "works hard"? =David does=the subject. Beer and wine are my favourite drinks. What "are my favourite drinks"? Beer and wine are=the subjects. The subject(s) of a sentence will answer the questions, "who or what." Clause

It is a group of related words, but unlike a phrase, a clause has a subject and predicate. Dependent Clause

Dependent Clause is a sentence that cannot stand alone. e.g. : When I went to store... Independent Clause

It’s a stand alone sentence. e.g.: I went to store Independent Marker

A connecting word used at the beginning of an independent clause. e.g.: Jim studied in the Sweet Shop for his chemistry quiz; however, it was hard to concentrate because of the noise. Independent makers:- also, consequently, furthermore, however, moreover, nevertheless, and t herefore Dependent Marker

A dependent marker word is a word added to the beginning of an independent clause that makes it into a dependent clause. e.g.: When Jim studied in the Sweet Shop for his chemistry quiz, it was very noisy. When is the Dependent Marker Dependent Makers-after, although, as, as if, because, before, even if, even though, if, in order, to, since, though, unless, until, whatever, when, whenever, whether, and while Rules:

i.

<Dependent sentence or clause><comma><Independent Clause> Or <Independent Marker><independent clause><independent clause> Eg: when I went to the store, I did not buy a bread ii. <Independent Clause><full stop><Independent clause> iii. <Independent Clause><semi colon><Independent clause> <Independent Clause><comma><conjunction><Independent clause> iv. <Independent clause><;><Independent marker><comma><independent clause> v. <independent clause><dependent clause> Phrase 

A phrase (modifier after comma) usually modifies the subject of the earlier phrase or the preceding noun, which ever is appropriate. Its a group of related words that have no subject-predicate combination and cannot stand alone as a sentence. Absolute Phrase

A participial phrase does not modifying the immediately preceding word: Refer 1000SC-663 Example: Some of the tenth-century stave churches of Norway are still standing, demonstrating that with sound design and maintenance, wooden buildings can last indefinitely. The first portion before the comma is the independent clause followed b by y the portion portion in blue, an absolute phrase. An absolute phrase does not modify any word in particular. It modifies the whole sentence that either precedes or follows

it. Therefore, is notsuch the churches which demonstrate anything. Instead, it is the fact that they are still standing which demonstrate X. This website storesit data as Now, the absolute phrase in and of itself has a sub-structure which looks like this: cookies to demonstrating enable essential [...] that site with sound design and maintenance , wooden buildings can last indefinitely. functionality, as well as marketing,  ‘With sound design and maintenance’ is a prepositional phrase which if you remove, you are left with: [...] demonstrating that wooden personalization, and analytics. You buildings can last indefinitely. Try to read the second half of the sentence without the comma and you will wonder whether "wooden buildings can last indefinitely" is may change your settings atphrase. any time part of the prepositional It just does not sound kosher. or accept the default Additive Phrasesettings. An expression typically set off with commas, that while it seems to be part of the subject is not, and therefore does not change the number of the verb. e.g.: And, Along with, In addition to, as well as, accompanied by, together, with, including, only, and can form a compound subject. PrivacyThe Policy other additive phrases do not form compound subjects. e.g. Wilfred and John are going to the beach. Wilfred, along with John, is going to the beach Marketing Disjunctive Phrase

Personalization

e.g.: Or, Either-or, Neither-nor For disjunctive phrases with the use of or, either-nor, or neither-nor, find the subject nearest to the verb and make that verb agree in

Analytics

number with this subject. e.g.: Neither Wilfred nor his friends are going to work. Either his friends or Wilfred is going on a holiday.

Save either or neither Accept Allin a sentence alone (without or/nor), they are not considered to be part of a disjunctive phrase. In When are

http://www.totalgadha.com

 

  such cases, they are considered singular and take only singular verbs. Conjunction

A conjunction is required to join to independent clauses. Conjunctions are connecting words at the beginning of an independent clause. Watch out for sentences that have no logical connectors between two independent clauses. These sentences are termed as run-on sentences because they involve two independent sentences connected by nothing more than a comma. It can be corrected by adding a coordinating conjunction. e.g. (run-on sentence): I need to relax, I have so many things to do. e.g. (Fixing run-on sentence): I need to relax but I have so many things to do. I saw two movies this weekend; both of them were good. (Joining two sentences with a semi-colon) I saw two movies this weekend, both of which were good. (Using a relative pronoun (aka subordinating conjunction) to join two sentences) I saw two movies this weekend, and both of them were good. (Using a coordinating conjunction to join two sentences) e.g.: Jim studied in the Sweet Shop for his chemistry quiz, but it was hard to concentrate because of the noise. and, but, for, or, nor, so, and yet.

When two clauses are connected by a Subordinating conjunction one cause has to be an Independent one. Adjectives and Adverbs

Many adverbs are formed by adding ..ly. to the adjective. An adjective can only modify nouns or pronouns. An adverb can modify verbs, adjectives, another adverb, preposition, or a phrase. e.g.: Real, Really. Nice, Nicely. Slow, Slowly. Incorrect: Schumacher is a real good driver. The adjective ‘good’ modifying driver can only be modified by an adverb ‘real’ is an adjective and so should not be used. Correct: Schumacher is a really good driver. The adverb ‘really’ now m odifies the adjective ‘good’ correctly. Colon & Semicolon

The colon is used to equate two parts of a sentence where the second part is dependent on the first part. It is used primarily to introduce a list, introduce a quotation or formal statement, or introduce a restatement or explanation. Usually examples are stated after colon The semicolon is used to connect two closely related statements. Both statements must be able to stand alone as independent sentences. Incorrect: Andrew and Lisa are inseparable; doing everything together. Correct: Andrew and Lisa are inseparable; they do everything together Colon: You should be able to insert the word namely after the colon. Incorrect: I love listening to: classical, rock, and pop music. (Incorrect) Correct: I love many kinds of music: [namely] classical, rock, and pop. Gerunds   Gerunds

Gerunds are words ending in ’ -ing -ing.’ Remember to treat them as if they were nouns. Use the word like to compare them and give them a possessive noun or pronoun such as my .

Major Errors of GMAT English

Although there are thousands of rules of English grammar, GMAC tests only a relatively small n umber on the GMAT. In fact, six grammatical issues show up in approximately 80% of all the questions in Sentence Correction.Learn the Big Eight well before you move on to less-tested subjects. Spotting bad sentences is the key to doing well on sentence structure test questions.  

Pronoun Error

  Misplaced Modifier This website stores data such as Parallel Construction   cookies to enable site   Verbessential Tense   as Subject Verb Agreement Errors functionality, well as marketing,   Parallelism personalization, and analytics. You   Quantity words may change settings at any time   your Idiom or accept the default settings.

Rule 1: Pronouns

Privacy Policy

Pronouns must unambiguously refer to a specific noun and also must agree in number with that tha t noun. The relative pronouns (that, which, who, what, when, where) refer to the nearest antecedent noun.

Marketing

Ignore context when trying to determine if a pronoun is unambiguous. Pronouns are words that are intended to substitute for more Personalization specific nouns. The rule on ambiguity is strictly construed, so a pronoun is considered ambiguous if there is any chance it could refer to more than one noun

Analytics

Pronoun reference error- referring pronoun is not correctly placed. pla ced. For example:

In the sentence “Samantha and Jane went shopping, but she couldn’t find anything she liked.”, the pronoun “she” does not refer to a Save unambiguously. Accept person It is All difficult to understand that whether “she” is referring to Samantha or Jane.

http://www.totalgadha.com

 

  The correct form would be “Samantha and Jane went shopping, but Samanatha couldn’t find anything she liked.” Subject Object Possessive

I Me My Mine You You Your, Yours He Him His She Her Her Hers It It Its We Us Ours They Them Theirs Who Whom Whose In this sentence, there is no way to tell for sure what w hat the word ’he’ is supposed to be referring to. This is called ambiguity. GMAC will correct a pronoun error in one of two ways: either by using a different pronoun that eliminates the ambiguity, or by eliminating the pronoun and using a specific noun. Which of these answers corrects the ambiguity? Answer (B) corrects the problem by substituting the pronoun ’they’ for ’he’. This eliminates any ambiguity because ’they’ refers to both John and Tim

Indefinite Pronouns

An indefinite pronoun is one that is not specific about the thing to which it refers (no clear referent) All pronouns that end in one, body or thing are indefinite pronouns. E.g. Everyone, Everybody, Everything Anyone, Anybody, Anything Someone, Somebody, Something No one, Nobody, Nothing The following are also indefinite: Whatever, whoever, Neither, Either, Each, Every. All the indefinite pronouns are singular. For each/every, if they precede a noun, the verb will take on a singular form as well. E.g. Each of the students is allowed to go on the field trip. Every dog and cat has paws. However, when each/every follow a subject, it has no bearing on the verb form. E.g. They each are good soccer players. There are however 5 indefinite pronouns that can singular or plural depending on the su bject. They are: Some, Any, None, All, Most E.g. Some of my marbles are missing (subject: marbles, verb: are). Most of the students are tired. None of my money is missing. Each, other/another, one

When two persons are referred in a sentence using each , other should be used. In case of three people the usage is each-another . When one one  is used to refer a noun only one  one can refer in that sentence. Eg. : One can achieve succeed in GMAT when one works hard. Relative pronouns are often used incorrectly today. 1) Referring Referring to things or animals – that, which 2) Referring Referring to people—who, whom They you’re positive there is a referring noun. Today we often use “they” 3) T hey – be careful that you don’t use this unless you’re

to replace the use of a proper proper noun which it is not. It’s a Pronoun. Which "Which" can be used as a restrictive or non-restrictive clause. It is a relative pronoun and should have an antecedent. In non restrictive

clauses It refers to the closest noun. Its always preceded by a comma. However, it does not always refer to the immediately preceding

noun. Sometimes, it, such much as like an absolute phrase does, will refer to the previous sentence as a whole. This website stores data e.g.: My brother got 95% in his exam, which really surprised me given the amount of studying he put in. ‘which’ here does not refer to cookies to enable essential site subject of prepositional phrase ‘exam’. functionality, as well as marketing, From the bark the paper birch personalization, andofanalytics. You tree the M enomini crafted a canoe about twenty feet long and two feet wide, with small ribs and rails of cedar, which could carry four persons or eight hundred pounds of baggage yet was so light that a person could easily portage it may change your settings at any time around impeding rapids. or accept default settings. Herethe ‘which’ obviously doesn’t refer to cedar, rather, it refers to ‘canoe’. The noun that the non restrictive clause modifies doesn’t necessarily need to be immediately proceed the comma. The OG concept is that it cannot refer to a vague idea that is expressed in the entire sentence, and that it must point to a noun (again, not necessarily immediately before the comma). Eg.: "The earth is not flat, which had puzzled many people in the old days." is wrong

Privacy Policy

Who/ Whom

Marketing

You can tell when ‘who’ is more appropriate, and when ‘whom’ is more appropriate by changing the adjective clause into a free running sentence. If the free running sentence contains he, she or they, use who w ho Personalization e.g.: He had none of the appearance of a man who sailed before the mast. (He sailed before the mast)

Analytics

If the free running sentence contains him, her or them, use whom e.g.: A man stepped in on whom I had never set my eyes before. (I have never set my eyes on him before) Save Accept All e.g.: Who are you going to marry? I am going to marry he/she.. (Wrong)

http://www.totalgadha.com

 

  Whom are you going to marry? I am going to marry him/her.. (Correct) Whose

Whose relates to people or to things. You can tell when to use ‘whose’ by changing the adjective clause into a free-running sentence. If the free running sentence contains his, hers, its, theirs, use whose e.g.: I am walking beside my father whose name is Simon Dedalus. (His name is Simon Dedalus) Where 

 ‘Where’ is generally used to indicate a place. When

 ‘When’ is generally used to indicate time. That

 ‘That’ modifies the nouns. It refers to to the immediate previous noun in the previous clause. It is a restrictive clause and provides essential information about the subject of a sentence. e.g.: The big GMAT book that is kept on the table is good. Note: No comma is used! "that" here refers to "the particular (definite/fixed) book on the table" That can refer to singular or plural. e.g.: In good years, the patchwork of green fields that surround the San Joaquin Valley town bustles with farm workers, many of them in the area just for the season. In the above example, what that refers to depends on the verb that follows it. If the sentence reads, ’a patchwork of green fields that ‘surround’. Here, ’that’ refers to ’green fields’. If the sentence reads, ’a patchwork of green fields that surrounds’. Here, ’that’ refers to ’patchwork’. Possessive Noun & Pronoun Reference

GMAT frequently tests pronoun reference in convoluted sentences in which pronoun is referred to possessive form of noun. The pronoun should have logical noun reference. e.g.: The department of labour allows the investment officers’ fees to be based on the performance of the funds they manage’. In the sentence above, ‘they’ can’t refer to ‘officers’. ‘Officers’ is used as a possessive noun to modify ‘fees’. Whereas, in the sentence below, ‘they’ refers to ‘officers’, ’The department of labour allows the fees of investment officers to be based on the performance of the funds they manage" Rule 2:-Misplaced Modifier (modifiers must stay close to home)

Sentences that begin with a verb, adjective+verb, and adjective phrases need to be followed by the noun or pronoun they are modifying. Usually end with –ing. Example: “Coming out of the department store, John’s wallet was stolen.”  “Coming” is the modifier. Was john’s wallet coming out of the s store? tore?  Incorrect  John’s wallet was stolen when he was coming out of the store. Possible solution to look for: i)  Correct the reference ii)  Put a noun or pronoun into the 1st part of the sentence turning the 1st part into an adverbial clause. Thus can stand apart without needing to watch the modifier.

A modifier, or modifying phrase, describes someone or something in the sentence. Modifiers are usually (not always) set off from the This website stores data such as rest of the sentence by commas. If the noun that is being modified by a modifier or cookies to enable essential site modifying phrase is not in the sentence, we have a dangling modifier. In some cases, the modified noun is in the sentence called a functionality, as well as marketing, but is not directly next to the modifying phrase. This is misplaced modifier. A modifying phrase should not be separated from the noun it personalization, and analytics. You modifies. Descriptive phrases must be placed directly next to what they modify. GMAC loves to start may change your at any time off sentencessettings with introductory descriptive phrases set off by a comma that are not followed by whatthe they modify. If the phrase itself is underlined, GMAC often corrects the error by or accept default settings. converting it to a clause. If the rest of the sentence is underlined, GMAC often corrects the error by moving whatever being modified so that it is directly next to the modifying phrase. Misplaced modifier  modifier  Privacyproblems Policy typically present you with a sentence that begins with a descriptive phrase set off comma. Sometimes the second part of the sentence is underlined as in this example For this sentence, if you ask yourself, ’Who was running down the street?’, then you can Marketing to see what’s wrong with the sentence as written.Answers (B) and (C) put the word ’I’ next ’running down the street’ and correct the misplaced modifier problem. Personalization Answer (C) doesn’t make any sense; therefore,(B) is the credited answer.

is by a begin to

Analytics this rule about misplaced modifiers: Phrases that modify or describe Remember other parts of a sentence must be placed directly next to the part of the sentence that they modify. Some types of misplaced modifier problems underline the introductory phrase instead of the second part of the sentence, as in this Save Accept All example.

http://www.totalgadha.com

 

  Since you cannot move the modified part of the sentence closer to the modifier (because modified part, cocaine derivatives, is not underlined), you must select an answer that changes the modifier so that it no longer violates the misplaced modifier rule. (A), (B), and (C) all repeat the original misplaced modifier error. Notice that (D) and (E) both correct the original error by re-wording the modifying phrase into a clause. The choice between (D) and (E) is tough.(E) is the correct answer. (D) is wrong because ’they were selling them’ contains a pronoun reference error. ’The government, a singular noun, ’ cannot be a ’they’.

the

You can change a misplaced modifier into a legal sentence by changing a phrase into a clause. “While leaving the bank, Evelyn’s purse was stolen.” (Incorrect, underlined portion is a  phrase) “As she was leaving the bank, Evelyn’s purse was stolen.” (Correct, underlined portion is a clause) Adverbial Modifier

When the word being modified is not a noun, the modifying phrase is called an adverbial phrase and does not need to touch the word being modified. e.g.: The running back ran towards the end zone, faster and harder than he had ever run before. The modifying phrase, faster and harder than he had ever run before modifies how the running back ran. Thus the phrase modifies  ‘ran’ and not ‘running back’. Modifiers with relative pronouns

Modifying phrases are often introduced by relative pronouns such as: which, that, where, who, whose, whom. On the GMAT, it is sometimes preferable to insert a modifier using a relative pronoun and a simple verb tense than using just an .-ing. form of a verb. e.g.: We test-drove a car having engine trouble (Awkward and not preferable) We test-drove a car that had engine trouble. (Relative Pronoun + Simple Verb Tense) Essential vs. Non-Essential Modifier

 ‘Which’  is  is used to introduce non-essential modifiers. These are clauses that provide information about a noun that is not necessary for identifying that noun. ‘Tha’ . is used to introduce essential modifiers. These are clauses that provide information about a noun that is nece ssary for identifying that noun. e.g. (Non-Essential): To find my house, walk down the left side of the road until you reach the third house, which is red. The sentence above always leads you to the third house on the left side of the road, and a nd this house happens to be red. e.g. (Essential): To find my house, walk down the left side of the road until you reach the third house that is red. The sentence above leads you to the third red house on the left side of the road.This may be the third house on the left side of the road, or it may be the tenth house on the left side of the road. That Vs Which

Most often than not, in GMAT, which would be preceded by a comma in the sentence. e.g. Get me the book, which is mine. Which is used to qualify the book i.e. which is mine. There may be many books in the room, but I want my book.  ‘Which’ should always refer to a noun. E.g. Get me the book, which is mine. So, ‘which’ here refers to the no noun un ‘book’.

This website stores data such as   Which should apply cookies to •enable essential site to things. •  The other thing--which must replace a noun, not a sentence or idea. functionality, as well as marketing, personalization, analytics. You e.g. Sales ofand United States manufactured goods to non-industrialized countries rose to $167 billion in 1992, than theyour previous year at and largely may change settings any timeoffsets weak demand from Europe and Japan. or accept the default settings. which is 14 percent more than the previous year

which is 14 percent more

which is 14 percent higher than it was the previous year 14 percent higher than the previous year's figure an amount that is 14 percent more than the previous year was wa s Privacy Policy an amount that is 14 percent higher than the previous year's figure B is incorrect, because which should refer to a specific noun in the preceding clause. That noun does not exist ( Rise of sales). So, it is replaced by an amount that is 14 percent higher … Marketing So, E is the correct answer

Personalization

 ‘That’ is a restrictive clause while ‘which’ is a non restrictive clause.

Analytics GMAT almost always (I say almost always because I've seen two questions that did not follow this rule, but the rule was violated in all h . In other words, if you see which without a comma before it, it's five answer choices) wants you Save Accept All to put a comma before w h i c h. probably wrong.

http://www.totalgadha.com

 

  An Example Both of these sentences are correct in GMAT land: •  Please go into the room and get me the big book, which is mine. •  Please go into the room and get me the big book that is mine. Yes, in GMAT land, these two sentences have two different meanings. Both of the following sentences would be incorrect in GMAT land: •  X Please go into the room and get me the big book which is mine. X •  X Please go into the room and get me the big book, that is mine. X Notice the commas--that's what makes all the difference. The Explanation 

Okay, we have in English this weird idea that we need to use different grammar in an adjective clause (a.k.a. relative clause) depending on whether the information in the adjective clause is necessary to specifically identify which noun we are referring to. For example, imagine you have one sister, and you are telling a friend that your sister is coming to visit you. Since this person is your friend, we can presume that he knows that you have only one sister. You utter a sentence like this to your friend in GMAT land: •  "My sister, who just graduated from college, is coming to see me." In GMAT land, since your friend (we presume) knows you well and knows that you have only one sister, this extra bit of information is considered unnecessary to identify which sister it is you are talking about. It is a sort of "by the way" information--"My sister is coming to see me, and oh, by the way, she just graduated from college." Now imagine you have two, three, or even more sisters. Let's imagine that one is a college professor, another is a webmaster, and this one who is coming to visit you just graduated from college. If you're talking to your friend, and you say only "my sister," and you do not mention her name, your friend might not know which sister you are talking about. So you add that extra bit of information--my sister who just graduated from college--to identify which sister it is you are referring to. In this situation, we have just correctly employed a very important grammar rule. So, if the person you're talking to, or the person who's reading what you've written, needs that extra bit of information to know which noun you're referring to, we say that that extra information is non-restrictive. This word doesn't really describe the function clearly; so many teachers say that this information is "extra." On the other hand, if you need that information to know which noun you are talking about, we say that the information is restrictive.  Again, this word is not really a good choice for clarity, and many teachers use the term "necessary information" instead. Finally, just to make English a bit more difficult, there is a rule that says that we should use a comma before or after ‘extra information clauses and phrases’, but not with ‘necessary information clauses or phrases’. phrases’. The idea here is that the comma represents the slight pause in speech or change in intonation that a native speaker might use when making such an utterance. Both that  and  and which are relative pronouns, i.e., they are grammatically the same, but their meanings are slightly different. Now, let's return to our original example sentences: Please go into the room and get me the big book, which is mine. In this sentence, the clause which is mine is "extra" because the information "the big book" is enough to identify which book it is that you want. We can assume that there is only one big book in the room. •  Please go into the room and get me the big book that is mine. In this sentence, the clause that is mine is "necessary" because the information "the big book" is NOT enough to identify which book it is that you want--it is probably the case that there are several big books in the room, so I need to add the information "that is mine" to identify which book it is that I want. More Examples • 

• 

I met with Bill Clinton, who is a lawyer. The name Bill Clinton is enough to identify which person I'm talking about-- who is a lawyer  is  is therefore extra information. I met with the man who is a l awyer. In this case, ‘the man’ is not enough information to identify which person I'm talking about-- who is a lawyer   is is

therefore necessary This website stores data such asinformation. •  The Sun, which is the only star in our solar system, is the source of heat for Earth. cookies to enable essential site Again, the name ‘the Sun’ already clearly identifies the noun; therefore, the information in the adjective clause functionality, as wellisas marketing,  ‘which the only star in our solar system’ is extra. personalization, and Youcentre of our solar system is called what? staranalytics. that is at the •  The In this case, since we time don't have a name here, we don't know which star it is that we are referring to. Therefore, may change your settings at any the information in the adjective clause ‘that is at the center of our solar system’ is necessary. or accept the default settings. ‘W hich’ requires a comma before it.  However, GMAT is very tricky at times, and can of course create a question in which this rule does NOT apply. In some questions in that ; they all used which without a comma.

PrivacyGMAT, Policy ‘which’ without a comma is used, but ALL the answer choices violated the rule. i.e., none of the answer choices used Marketing

Here's an example of what I mean:

Personalization

Lucise in San Francisco is a place which anybody can visit. visit .

Analytics (A) which anybody can visit (B) which any person could visit (C) which no person could not Save Accept Allvisit (D) which, if they wanted to, any person could visit

http://www.totalgadha.com

 

  (E) which any person could visit if they so desired The whole point of GMAT sentence correction is that we must choose the best answer, NOT the perfect answer. A modifier introduced by ‘which’ can be removed from the sentence without the sentence losing any essential meaning. Whereas a modifier introduced by ‘that’ is essential to the meaning. Commas are used to separate non-essential modifiers from the noun that is modified. The pronoun ‘who’ can be used in either essential or non-essential modifiers. e.g. (Essential): Only guests who are accompanied by tenants may use the gym facilities. The sentence above identifies a subgroup of guest to whom the pool is open: those accompanied by tenants. e.g. (Non-Essential): Only guests, who are accompanied by tenants, may use the gym facilities. The sentence above indicates that only guests (as opposed to tenants) may use the gym facilit ies and that they just happen to be accompanied by tenants. Exercise Identify the Correct Modifiers

i. Although taken largely for granted in America, the Chinese are seeing the debut of nationwide broadcasting only now. ii. Once the dominant world power, Great Britain has seen its colonial holdings and international prominence shrink concurrently throughout the twentieth century. iii. The irritation of the stomach caused by aspirin can be avoided if the aspirin tablet is given a coating that will not dissolve until the tablet reaches the intestine. iv. Based on a comprehensive study done in the 1980s, nutritionists have made strong recommendations about the percentage of our daily calories that should come from fat, although they do make some distinctions between animal and other kinds of fats. v. Although born poor and with virtually no social connections, the remarkable rise of John J. McCloy to the position of ultimate insider took only 10 years. Answers

i. Yes. ’The Chinese’ were not ’taken lately for granted in America.’ The introductory phrase is intended to modify ’nationwide broadcasting’. ii. No, this sentence does not contain a misplaced modifier because the descriptive phrase (’once the dominant world power’) is next to what it modifies (’Great Britain’). iii. No, this sentence does not contain a misplaced modifier because it does not have a descriptive phrase separated from something that it modifies. iv. Yes, this sentence contains a misplaced modifier because ’nutritionists’ were not ’based on a comprehensive study done in the 1980s’. The sentence could be corrected by starting it with ’relying on a comprehensive study done in the 1980s’. v. Yes, there is a misplaced modifier here because the opening phrase (’although born poor and with virtually no social connections’) is not next to what it modifies (’John J. McCloy’) Subjunctive

The subjunctive is a voice that follows very specific rules.A verb is in the subjunctive mood when it expresses a condition which is doubtful or not factual. For hypothetical situations, be sure to use both a ’were’ and a ’would’. For a demand or recommendation , make sure you use the infinitive form of a verb without the word ’ to to.’ Refer SC-1000-636 E.g. several senior officials spoke to the press on condition that they not be named in the story. There is an uncertainty of what the press will do. Obviously the sentence tests for subjunctive mood. Since it is uncertain the usage of  ‘will’ is wrong Subjective’Verb’

There are 2 forms it takes up. " <subjunctive verb> + Noun + < infinitive>. Or "<subjunctive verb>+that + Noun". Note that there is

This website stores dataAlthough such asboth are grammatically correct, later form is always preferred over former. no .to. in the later. Remember of .should  . following subjective verb is always wrong. cookies to enableuse essential site e.g.: Form 1: requires that …. functionality, as well as marketing, e.g.: Form 2: requires... personalization, and analytics. You ask, e.g. of subjective verbs: advise, a sk, arrange, better, demand, desire, direct, imperative, pray, order, propose, recommend, request, suggest,your insist and urge. may change settings at any time or accept the default settings. Subjective if & non if According to traditional rules, you use the subjunctive to describe an occurrence that you have presupposed to be contrary to fact. It takes up the form, if ...were ... would. e.g.: If he were sorry, he would have apologized by now. PrivacyI wish Policy she were not going away. She’s already acting as if she were going to be promoted. Marketing Suppose she were to resign, what would you do then? If-Then Construction Personalization

Sentences Analyticsthat use the word ‘if’ to describe hypothetical conditions require a conditional verb construction. These sentences have two parts: if clause, and the then clause. The word ‘if’ does not always signal a conditional sentence. Only when the sentence has ha s a ‘then’ clause, then the sentence is considered a conditional sentence. Also note would/could w ould/could never appears in the ‘if’ clause.The actual word then the GMAT Test. Saveis frequently omitted AcceptinAll

http://www.totalgadha.com

 

  If Clause Then Clause

Present Tense Will + Base Verb Past Tense Would/Could + Base Verb Past Perfect Tense Would/Could + Have + Past Participle If v / s Whether

 is used to introduce the first of two or more alternatives and sometimes repeated before the second alternative. ‘Whether’ is ‘Whether’  is preferred over ‘if ’ when a future possibility is stated. Usage of ‘ whether or not’ is almost wrong on GMAT Land e.g.: Incorrect: I do not know if I will go to the dance Correct: I do not know whether I will go to the dance It does not matter to me whether we buy the car or lease it whether John decides to go to the party or whether he stays at home is not the issue. Incorrect: Her client didn’t tell her if he had sent his payment yet. Correct: Her client didn’t tell her whether he had sent his payment yet. Because v / s Due to

 ‘Due to’ means ‘caused by’ It should should only be used if it can be substituted with ‘caused by’. It is used in situations where it means  ‘attributable to’. ‘Because’ is used to state a reason. Possible structures: ‘Due to + Noun Phrase’ and ‘Due to the fact that + Main’ Clause., and .Because + Main Clause. and .Because of + Noun Phrase.. e.g.: SC-1000#504 (Because Vs due to) SC-1000#879 (Because Vs due to) SC-1000#990 (Usage of "due to") Rule 3: Parallel Construction

Sentences containing a list or a comparison must follow the rule of parallel construction. First First,, every item in a list must be treated similarly. Look for an item in the list that is not underlined to determine the proper form. Second, Second, items being compared must be of similar type. Apples must be compared to apples and oranges to oranges. Often, this means that nouns must be compared to nouns and action words to action words here are two common errors that come into play when making sure one part of a sentence is parallel to, or in balance with, the other parts. Lists The Rule:  Rule: All items in a list must be treated similarly. In this example, the list consists of ’to work, play, and to go’ . The items in the list are not treated the same Examine the answers.

In which answer are all the items treated similarly? In answer (B), all the items in the list are treated the same. Note that if the sentence had read, ’Floyd wanted to work, to play, and to go to the store’ , this would also be grammatically correct.  Lists are not always easy to spot. Make sure to take the time to identify all the elements in a list. Sometimes you have to read a sentence in a different way to correctly identify all the elements. items in the list must be parallel and there must be an "and" before the last item. If the list does contain "and" , it’s incomplete hence wrong. If the list contains more than one "and", then it’s a trapped list. However there can be multiple ands when items are different.

This website stores data such as cookies to enable essential site functionality, as well as marketing, personalization, and analytics. You may change your settings at any time or accept the default settings.

Privacy Policy Marketing

e.g.: Agrarian revolution involved the large-scale introduction of enclosed fields and of new farming techniques and crops, and the substitution of commercial for subsistence farming. Personalization e.g.: Some bat caves take on different duties such as defending the entrance, acting as sentinels and sounding a warning at the approach of danger, and scouting outside the cave

Analytics Pronouns

Often, pronouns such as .which., .that., .those., .who., etc. - signal parallel structures. If one item includes a pronoun, it is often appropriate to include the same pronoun in parallel items. SaveI prefer to hireAccept All who work hard to those who don’t. e.g.: employees

All not list

http://www.totalgadha.com

 

  I enjoy going out with people who are humorous than those who aren’t. Exceptions in Parallelism

However there are some exceptions in parallelism which should not be confused with parallelism. Two gerunds + noun constructing is valid. e.g.: He liked sailing, swimming and girls. Verb + Gerund

e.g.: Some bat caves act as sentinels, sounding a warning at the approach of danger, and scout outside the cave Split Infinitive

The infinitive is the ’to’ form of a verb. When a verb takes the form to + the verb, it is called the infinitive form. Avoid sentences that insert a word between to and the verb. This error is called a split infinitive and is often incorrect. Don’t split it. Don’t put anything between ’ to to’ and the verb e.g. Correct: I need you to run quickly to the store. Wrong: I need you to quickly run to the store. Wrong : I cycled down to beach and swimmed. Correct: I cycled down to beach to swim. There are two kinds of ERS sentences that test the parallel construction. The first is first is a sentence that contains a list, or has a series of actions set off from one another by commas. The second kind second kind is a sentence that’s divided into two parts. Both types must have parallel types of verbiage:   …..to eat, to sleep ate _____, slept ____, drank ____.   Bad construction might look like: …to eat, sleep   …ate _____, sleep _____, drank ____.  

There are a few more things to be observed in a parallel construction sentence. Comparisons must be logical and compatible

Find the two things being compared and see if the sentence is structured in balance. Don’t be afraid to consider changing verbs or adjectives to get the balance. The words "like," "unlike," "similar to," "as…so", “when” and "in contrast to" are the most common indicators of comparisons. In comparisons, compatibility is determined by subject matter . For example:  “As domesticated animals, indoor cats typically lose their ability to hunt for their own food, so too do domesticated dogs c come ome to rely exclusively on their owners for sustenance.” Here, domesticated cats are compared to domesticated dogs, and the comparison works because they are both domesticated animals — they are like terms. Whenever you see a comparison being set up in a sentence, check to see that the terms of the comparison are compatible. Parallelism is not just about clauses, but verb usage

Example: -ing and –ing, to…… tto…… o…… , either ….. or, neither ……. nor. In a series of two or more elements, what you do on #2 determine s what you do on 3+. In other words, everything after #2 must match #2: •  I like to swim, to run, and to dance. •  I like to swim, run, and dance. are okay. •  I like to swim, run, and to dance. This website such as and dance. are NOT okay. I like todata swim, to run, •  stores cookies to enable essential site This error is not a frequently encountered error, but it is worth knowing and practicing such errors. In such sentences, generally functionality, as well as marketing, two things or items are compared. personalization, andthe analytics. When sentenceYou compares two items. Ask yourself, can they be really compared? When the sentence may change your settings at anycompares time two actions as well. or accept the default Usually, the settings. problem is with hidden comparison where two things or actions are compared, but another two items or actions are a re

intertwined and you lose the comparison relationship. Example: “Synthetic oils burn less efficiently than natural oils.”

Privacy Policy The sentence is wrong because we are trying to compare compare how well each oil burns and not the oils themselves.

But do you see how

the actual thing being compared is easily missed? Correct- Synthetic oils burn less efficiently than natural oils do. Marketing

Personalization Rule 4: Verb Tense

Analytics On the GMAT, tense problems are often just a matter of parallel construction. In general, if a sentence starts out in one tense, it should probably remain in the same tense.

Save Accept of Alltense: You don’t need to memorize types. Just be familiar. Some major categories

http://www.totalgadha.com

 

  a.  b.  c.  d.  e.  f. 

Present tense  Simple Past  Present Perfect  Past Perfect  Future  Present Perfect 

g.  Past Perfect  h.  Present Progressive  i. 

Perfect Progressive 

example: He walks three miles a day. example: When he was younger, he walked three miles a day. example: He has walked. example: He had walked. example: He will walk. Describes action that began in the past but continues until the present. Key identifier – “has” “have”. Note: Sometimes used when deadline exists. Describes action that started and stopped in the past. Key identifier – ‘had’  Used as emphasis by the speaker that the action is happening this very minute. Key identifier – verb ‘to be’ + a verb with an-ing ending.  Occupies more than one moment in the past.In other words, ongoing for a period of time. Key identifier – “had been” 

One exception to this rule is a sentence that contains the past perfect (in which one action in the past happened before another action in the past). Examples:   He had ridden his motorcycle for two hours when it ran out of gas. The dinosaurs are extinct now, but they were once present on the earth i n large numbers.   Two events that have taken place, are taking place or will take place at the same time must have the same tense in the sentence. Passive verbs begin with the form of ‘to be’ (Example: to be, were, was) and end with a different verb in the past tense. Use the present perfect perfect (has or have) when an activity began in the past and continues uninterrupted to the present. Use past perfect (had) when an activity began in the past but is then interrupted by another action in the past. Some sentences will contain errors in their verb tense. Although there are many verb tenses in English language, GMAC has chosen to test only a few. Just remember that tense refers to time and always look for time clues in a sentence to let you know when something is supposed to have happened. Generally, if a sentence starts in a particular tense, it should stay in that tense. perfect  is used for actions that began in the past and continue to the present. The present perfect Use ‘has’ for singular subjects and ‘have’ for plural subjects. In each of these examples the implication is that the action is still continuing.

the

Future tenses are not used in ‘before’ clauses. Could is a modal expressing past (potential) ability. If something is assumed in the past the usage of .Could. is appropriate. It is used to refer to Future in the past. If it is ongoing, ‘c ould’  cannot  cannot be used. Past perfect  perfect is used for actions that start in the past and are interrupted by another action also occurring in the past. Use ‘ had’ for both singular and plural subjects. On the GMAT, the past perfect is only appropriate when a sentence makes clear that there was an interrupting action and that the original action is no longer occurring. Here, ’until it jammed’ makes clear that the copy machine is no longer working. Some Past Participle of Irregular Verbs

This website stores data such as cookies to enable essential site functionality, as well as marketing, personalization, and analytics. You may change your settings at any time or accept the default settings.

Privacy Policy Will/ Shall

Marketing

‘Shall’ expresses simple futurity, while ‘Will’ expresses determination. But which word expresses which meaning depends on whether you’re using first person (I, we), on the one hand, or second (you) or third person (it, they), on the other. In Personalization the first person, ‘shall’ expresses futurity, and ‘will’ expresses determination. e.g.: I shall do it tomorrow. I will succeed, even if it’s the last thing I do. Analytics

In the second and third persons, it’s the opposite. ‘Shall’ expresses determination, and ‘will’ expresses futurity. e.g.: You shall succeed, even if it’s the last th ing you do. They will do it tomorrow.

Save

Since

Accept All

http://www.totalgadha.com

 

  Since should have a present perfect tense. Like vs. Such As Question: What's the difference between like and such as?

Can you buy me some fruit like oranges or grapefruit? How the GMAT Official Guide would explain this mistake : Using like in this answer choice mistakenly suggests that the utterer of

the request does in fact not want oranges or grapefruit, but rather some other kind of fruit that is similar to oranges or grapefruit. In normal English: In GMATLand, like means similar to, and such as means for example. Take a look at these examples: •  Can you buy me some fruit like oranges or grapefruit? In GMATLand, this sentence would mean that you do NOT want oranges or grapefruit; instead, you'd prefer some fruit similar to oranges and grapefruit. For example, you may want pomelo, lemons, or limes. Yes, I know this sounds a little crazy, but our goal is to understand what GMAT is looking for, not what is " correct" English. •  Can you buy me some fruit such as oranges or grapefruit? Yes, this is what we're supposed to say in GMATLand -- oranges and grapefruit are exam ples of the type of fruit we want. I would like you to buy such fruit as oranges and grapefruit for me, if you don't mind. This is simply a variation -- notice how such and as are separated. Separating the two elements tends to make this pattern a bit harder to see. Exercise

i. Just as I crossed over to the dark side, ______ will you, my son. ii. The mule, _____ the donkey, is a close relative of the horse. iii. Many of my favourite ice cream flavours, _____ chocolate chip and strawberry, are also available as frozen yogurt. iv. Her coat is just _____ mine. v. He did not vote for Ralph Nader, _____ I did. vi. Aimee said he wasn’t coming, and Luis said he was, so now I don’t know_____ he’s coming. vii. What I do know is that _____ he comes, he’ll bring a nice bottle of Rioja. viii. His friends do not believe the ring he bought at the auction _____ J ackie O’s; they all think he was tricked. Answers :

i. so too ii. like; ‘like’ means ‘similar to.’ iii. such as; "such as" means ‘for example.’ iv. like; use ‘like’ when comparing nouns v. as; use ‘as’ when comparing noun/verb combinations vi. whether; use ‘whether’ when choosing between two options. Avoid the redundant phrase "whether or not". vii. if; use "if" when dealing with a conditional statement viii. to be; while you can believe that something is true, you believe something to be true Subject-Verb Inversion

There are at least eighteen types of inversion:  inversion:   1.  negative intro 2.  intro adverbial (in, down, prepositional phrase) 3.  intro -ed 4.  comparative 5.  stores intro comparative This website data such as 6.   as cookies to enable essential site 7.  as so... that... functionality, well as marketing, 8.  had, should, were You personalization, and analytics. may change your settings at any time 9.  there is or accept the 10.  default here is settings. 11.  intro -ing 12.  emphasis

Privacy Policy

13.  the bigger, the better

14.  questions Marketing 15.  "story speech"

Personalization 16.  nor

Analytics 17.  so do I/neither do I 18.  intro adjective

Save

Accept All

http://www.totalgadha.com

 

 

Type

Examples

Notes

1. neg. intro

Never do I sleep.

Question form is obligatory.

Only at night can I study.

Used with all verbs.

In no way could I help you with your Japanese

This one is very common on the TOEFL and somewhat

grammar question.

common on the GMAT and GRE.

I believe that only rarely will I need your help.

We need to learn the various types of words and phrases

Not until I got home did I realize that my shoes

that require this type of inversion.

were untied.

Notice that sometimes the inversion occurs right after the neg intro form and sometimes it occurs in the next subject and verb. See Neg Intro for more info.

2. intro adverbial

Into the room ran the lady.

Inversion is optional.

First comes love, then comes marriage.

Used with be-verbs, linking verbs, and a nd verbs of direction.

After A comes B, then comes C, next comes D.

This one is less common on the TOEFL, but more common

Down came the rain and washed the spider out.

on the GMAT and GRE. Notice that sometimes we have an adverb, like first and down and sometimes we have an adverb phrase like into the

This website stores data such as cookies to enable essential site functionality, as well as marketing, personalization, and analytics. You may change your settings at any time or accept the default settings. 3. intro –ed

room or after A. These adverbs and adverb phrases usually show location or direction. This type of inversion usually only occurs with be-verbs, linking verbs and verbs that show direction or movement, like come, go, run, etc.

Found in San Francisco is Lombard Street, the so-

Inversion is obligatory.

called crookedest street in the world.

Used with be-verbs.

Lost among the old tables and chairs was the

This one is very common on the TOEFL, GMAT, and GRE.

Marketing

priceless Victorian desk.

This type of inversion usually occurs with be-verbs, but

Personalization

Located between San Francisco and Marin County

sometimes with linking verbs.

is the Golden Gate Bridge.

Notice that the phrase is the complement of the be-verb.

Privacy Policy

Analytics Save

Accept All

http://www.totalgadha.com

 

  Type

Examples

Notes

4. comparatives

Cheetahs run faster than do antelopes.

Inversion is optional.

You speak Chinese better than do I.

Used with all verbs.

Jessica is more interested in Computer Science

This form of inversion is common on the TOEFL, GMAT, and

than is Benjamin.

GRE. We normally only have inversion here if we are comparing subjects of the verb, not objects. For example, in the following two sentences, we are comparing objects, carrots and potatoes, not the subject I.: ϑ ϑI

like carrots more than I do potatoes.

Λ ΛI

like carrots more than do I like potatoes.

Now, in this sentence, we are comparing subjects, I and my friend Carl: ϑ ϑI

like carrots more than does my friend Carl.

5. intro

Bigger than an apatosaur is the blue whale.

Inversion is obligatory.

comparative

More important than your personal statement is

Used with be-verbs.

your GPA.

This form is more common on the GMAT and GRE than it is

No less impressive than the invention of the laser

on the TOEFL.

was the development of the wheel.

Notice that we can only use this form of inversion when the verb is a be-verb since in every case, the comparative is the complement of the be-verb. Remember that less than is also a comparative.

6. as

Megumi is from Japan, as is Sato.

Inversion is obligatory.

So-eun wants to leave early today, as does Oi.

Used with all verbs.

If thrown into the water, camels can swim, as can

We can only use inversion if we are using as for

cats.

comparisons. as is one of the trickiest words in English; it can have many ma ny different meanings.

7. so… that…

So happy was I that I bought flowers for

Question form is obligatory.

everybody in class.

Used with all verbs.

quickly This website stores data So such as did she leave that we did not even cookies to enable essential sitewas gone. realize functionality, as well as marketing, So rarely does a comet appear visible to the personalization, and analytics. Youthat when one does, it is considered a naked eye may change your settings at any time major event. or accept the default settings.

Privacy Policy Marketing Personalization Analytics Save

Accept All

This is not so common on the TOEFL, but is fairly common on the GMAT and GRE. The so… that… clause must before the verb in for this type of inversion.

http://www.totalgadha.com

 

  Type

Examples

Notes

8. had, should,

Had I remembered Tomomi’s birthday, she

Inversion is obligatory.

were for if -

wouldn’t be mad at me now.

Used with all verbs.

clauses

Should you need a hand, I will be more than

This is somewhat common on the TOEFL and more common

happy to help you.

on the GMAT and GRE.

Were I you, I think I would study more for your

This type of inversion is kind of special. Notice that we can

exam tomorrow.

only use this type of inversion when we are using an ifclause. In other words, if is omitted: even though the word if does not appear in the clause, we still have the meaning of an if-clause. For more information, see had, should, were.

9. there is, there

There is a good restaurant nearby.

Inversion is obligatory.

are, there exists,

There comes a time in every person’s life when

Usually used only with these verbs.

there comes, etc.

she realizes that she is responsible for her own

This form of inversion is common on the TOEFL, GMAT, and

happiness, not other people.

GRE, as well as in spoken and written English.

Scientists hypothesize that there exists a certain

Most people remember there is and there are. BUT we must

type of particle that can travel faster than the

also remember that there are other verbs that we can use

speed of light.

instead of is and are. The most common ones are exist, come, and go.

10. here is, here

Here is some good food for you to try.

Inversion is obligatory.

are, here comes,

Here are the books that I don’t need anymore.

Usually used only with these verbs.

here come

Here comes the bus!

You will probably not see this on the grammar section of the TOEFL or on the GMAT or GRE. It could, however, appear on the Listening Comprehension Section of the TOEFL. We use this form mostly in spoken English.

11. intro -ing

Burning out of control was the forest located in

Inversion is obligatory.

the foothills of the Sierra Nevada mountains.

Used only with be-verbs.

Coming in last in the race was Joe “Elephant

This form is not common on the TOEFL, but might show up

Legs” Blow.

on the GMAT or GRE.

Not helping the situation was little Susie, who was

Notice the intro –ing phrase is the complement of the be-

This website stores data throwing such as newspaper on the spreading fire. cookies to enable essential site functionality, as well as marketing, 12. emphasis Boy am I hungry. personalization, and analytics. You Is it ever hot in here! may change your settingsDoatyou anyknow timehow to cook! or accept the default settings.

verb. Inversion is optional. Used with all verbs. You will probably not see this on the grammar section of the TOEFL or on the GMAT or GRE. It could, however, appear on the Listening Comprehension Section of the TOEFL. We use this form mostly in spoken English.

Privacy Policy Marketing 13. the bigger,

The closer an object is to another object, the

Question form is optional.

the better

greater is the gravity between the two objects.

Used with all verbs.

Personalization Analytics

Save

Accept All

http://www.totalgadha.com

 

  Type

Examples

Notes

14. questions

Is this the last example?

Inversion is obligatory.

Do you enjoy reading these lists?

Used with all verbs.

Are we finished yet?

You will probably not see this on the grammar section of the TOEFL (TOEFL doesn’t test questions anymore) or on the GMAT or GRE. It would, however, appear on the Listening Comprehension Section of the TOEFL.

15. "story

 “I think it’s time to go,” said Susan.

Inversion is optional.

speech"

 “It’s time for you, but not for me,” replied Gary.

Used with verbs that report speech.

 “Maybe we should collect our thoughts for a

You will probably not see this on the grammar section of the

16. nor

moment,” commented Lany.

TOEFL or on the GMAT or GRE.

No one has volunteered for the job, nor do we

Inversion is obligatory.

expect anyone to volunteer in the future.

Used with all verbs.

Hok-ming cannot speak Portuguese, nor can José

You might see this on the adaptive TOEFL if you are scoring

speak Cantonese.

high and it could appear on the GMAT or GRE.

The zoo regulations will not permit you to touch

Remember that nor is considered a conjunction, but we use

the animals, nor would most people advise you to do so.

it between two sentences (not between any two elements like the other conjunctions).

17. "so do I"/

 “So do I.”

Inversion is obligatory.

"neither do I."

 “So can Terry.”

Used with all verbs.

 “Neither do most people I know.”

You will probably not see this on the grammar section of the TOEFL or on the GMAT or GRE.

This website stores data such as cookies to enable essential site functionality, as well as marketing, personalization, and analytics. You may change your settings at any time or accept the default settings.

Privacy Policy Marketing Personalization Analytics Save

Accept All

http://www.totalgadha.com

 

  Type

Examples

Notes

18. intro

Beautiful beyond belief was my baby daughter.

Inversion is obligatory in most cases.

adjective

Happy about their acceptance into their dream

Used with be-verbs.

schools were Lany and Tomo.

This one is fairly rare and probably would not appear on the

Quick and painless will be your medical

TOEFL, but you might see it on the GMAT or GRE.

procedure.

Inversion is sometimes not used in poetic language.

What does Question form is obligatory  mean?  mean? This simply means that you MUST invert the subject and the verb in this construction. In other constructions, inversion is optional, but in these constructions, it is required. For example, you may say: •  She

runs faster than do most of her classmates.  classmates. 

(verb comes before the subject) or •  She

runs faster than most of her classmates do. 

(subject comes before the verb) BUT •  Never

have I heard such a thing!  thing! 

CANNOT become •  XX

Never I have heard such a thing! XX thing! XX  

Anyway, I think most people would say that this is simply a bad sentence and should be rewritten. This sentence is more of a grammar puzzle than a real sentence. Well, you can't go wrong if you write it in the singular, can you?   teacher Thestores together This website data such with as the student IS (or ARE) going to...?   The teacher and the student ARE (or IS)going to? cookies to Generally enable essential site speaking, we need a conjunction to create a plural subject from more than one singular noun. " together with" is NOT a functionality, as well as marketing, conjunction, and therefore cannot create a plural subject. "and" on the other hand, IS a conjunction and CAN create a plural subject. personalization, and analytics. You I'm concluding: may change your settings at any time or accept the default settings.

With plural nouns, use plural verbs:

Privacy Policy   One third of the students have graduate degrees.  

Fifty percent of the computers have CD-ROM drives.

  Many researchers depend on grants from industry. Marketing

With collective nouns, use either singular or plural, depending on whether you want to emphasize the single group or Personalization its individual members:

Analytics Half of my family lives/live in Canada.      

Save

All of the class is/are here. Ten percent of the population is/are bilingual.

Accept All

This is another reason, and this one's a bit harder to explain. In a nutshell, though, we can't use a that noun clause with the word

http://www.totalgadha.com

 

  directive, just as we cannot with order , as hello gmat has pointed out. Rule 5:- SUBJECT VERB AGREEMENT ERRORS

Subjects and verbs must agree in number. Always identify the subject and verb of a sentence. Eliminate any intervening phrases that are designed to distract you. Watch out for words that sound plural and are really singular, or vice versa GMAC will place irrelevant text, usually in the form of a prepositional phrase, in between the subject and verb of a sentence in order to distract you from the correct answer. This is GMAC’s favourite trick in the Sentence Correction section, so for the last time, be sure to isolate the subject and verb from any intervening information when you consider whether they are in agreement Pre-Exercise

i. The number of workers have/has increased steadily each year. ii. Neither the cost nor the selling price of the new product due to be introduced over the next two years has/have been determined as of yet. iii. The number of job offers a typical business school graduate receives stagger/staggers the mind. iv. Every Sunday, Bob and three of his college fraternity brothers goes/go to a neighbourhood pub and reminisce. v. The species of dinosaur known as the humongosaurus is/are among the largest creatures ever to walk the earth. vi. Each of her suitors plead/pleads with her on bended knee. Answers

Has, has, staggers, go (Compound subjects, like ’Bob and his friends’ require plural verbs), is (Species is singular), pleads (’Each’ really means ’each one’, which is a singular subject.) Approach Identify the Subject and Verb.

The first step in subject/verb agreement problems is to identify the subject and verb in a sentence. What do you think are the subject and verb in this sample question? Anything that Separates the Two Two..’Each’ is the subject of this sentence and ’were’ is the verb. Notice that the intervening prepositional phrases ’of the men involved’ and ’in the extensive renovations’ are irrelevant to the subject and verb. So, take them out and read what is left of the sentence Agreement.. Now that you have the correct subject and verb, make sure that they Check for Agreement agree: singular subjects must have singular verbs, plural subjects must have plural verbs. Is ’each’ singular or plural? Does it go with ’was’ or ’were’? Split  ’Each’ is singular and should be paired with ’was’. ’Each was en gineers’ is incorrect, so (E) is the correct Look for the 2/3 Split ’Each’ answer Either/Neither  Either/Neither 

When two distinct words or phrases are joined by the correlatives either… or, neither… nor, not only…. but also, the number (singular or plural) of the word or phrase nearest to the verb determines the number of the verb. Example: Either his parents or he is bringing it (notice “is” is singular) This can be a confusing sentence because   parents is plural, but we pay attention to he which is the noun “he” tells us that we ne ed to keep “is” singular. Example: Either he or his parents are bringing it. it. Notice “parents” is plural and is the closest to the verb so we use “are”   which is plural. ’Either’ and ’Neither’ both are singular when used as the subject of a sentence. For example: Neither  Neither of the boys has been arrested before. If a sentence is of the form ’either A or B’ or ’neither A nor B’ then match the verb to the second subject. were  happy on the first day of school. For example: Neither Joe nor his cousins were

This website stores data such as cookies to enable essential site The Number vs. A Number functionality, as well as marketing, As a stand-alone word and as a collective noun, "number" can take a singular or a plural form. personalization, and analytics. You e.g.Two hundreds persons were at the party; the number(s) is (are) just astounding. may change your settings at any time However, when preceded by an article and followed by preposition "of","number" is singular and the verb that follows "number" will be or accept the default settings. conjugated singular or plural depending on whether there is a definite or indefinite article in front. The expression ’the number of . . .’ is singular, while ’a number of . . .’ is plural. The following sentences are both correct:

PrivacyThe Policy number of bad movies showing this summer is number  is  unbelievable. number  of my friends are  are going to the beach this weekend A number

Marketing

"a number of ..." always takes plural verbs. "the number of ..." always takes singular verbs.

Personalization

Eg: The number of people has increased Analytics A number of people have gone The important thing here is that the number in the first example (the number of bad movies) is an actual number—1,000, for example. Saveif you add more Accept All to the original number, there will still be one number, right? Even bad movies The usage of "numbers" is also correct, and means that there are many people in that group. For example, it is correct to say:

http://www.totalgadha.com

 

  People are leaving California i n greater numbers. People are spending more money on the Internet in greater numbers. numbers . Pronouns   Indefinite Pronouns Plural and Singular

Once you start with one, you need to stay in the same quantity (singular or plural). Singular Pronouns (Memorize these)

Hint: Do you see the categories I setup? It’s SANE to memorize this Some Any No Every

Everyone Everybody Everything Anyone Anybody Anything

Someone Somebody Something No one Nobody Nothing

Either Neither

One Each

Whoever Whomever

His

Everyone in the senior class is  Everyone  is sick with the flu

 

Plural Pronouns (Memorize these)

Both Few  

Their Others

Many

Several

Singular and Plural Pronouns – depends on whether the noun is singular or plural (Memorize these)

Some

More

Most

All

Compound Subjects

The use of and usually signals a compound subject. This means you’ve got several people or things performing the action. The subject is plural. For example: The bride  bride and her bridesmaids were were  a nervous wreck

Collective Nouns

Nouns that represent a group of objects are usually singular, and require a singular subject. Collective nouns include words such as group, team, assembly, jury, team, country, family, species, and The French Antilles. They are plural when they act as individuals. e.g.: family, majority, team, audience, or anything that represents one group. celebrates  a victory with a group hug. My company’s softball team  team always celebrates "curfew" is a singular countable noun and therefore requires a determiner (the). Skill can be both a countable noun as well as a s a non-countable noun. It all depends upon the context. Have a look at the example below:

This website stores data such as 1. Harry knows quite a few driving skills. cookies to enable essential siteto ask Harry about his driving skills, I would ask…. Conversely, if I were functionality, as well as marketing, 2. How much skill do you have in driving a car, Harry? personalization, and analytics. You So you see, the word "skill" remains the same but depending on the context, skill can be a non-countable or a may change countable your settings noun?at any time or accept the default settings. Singular Subjects that look Plural.

PrivacySometimes Policy a subject has an ’s’ on the end, even though it

is really singular. Test this by asking yourself whether the subject is one thing or several. For example: Economics is is  one of my favourite subjects.  subjects.   Marketing When two nouns are in the sentence doing an action together but they are linked with i)  Along with Personalization ii)   Together with iii)  With Analytics iv)   As well as   In addition to v) vi)  Accompanied by

Save

Accept All

… this does not make the following action they do plural. Only “and” can take the two singulars and make their action plural. 

http://www.totalgadha.com

 

  For example: Janie, with her poodle limping behind her, walks to the dog park. Explanation: Janie is singular. The poodle is singular. singular. They both are doing the action together, but the use of “with” means that we need to keep the verb singular. “Walks” is singular and “Walk” is plural. Remember, a verb that ends with an –s is singular. Definite/ Indefinite articles

English has two types of articles: definite (the) and indefinite (a, an.) The use of these articles depends mainly on whether you are referring to any member of a group, or to a specific member of a group. Indefinite Articles: A Articles: A and AN signal that the noun modified is indefinite, referring to any member of a group. These i ndefinite articles are used with singular nouns when the noun is general. Article: A definite article is used before singular and plural nouns when the noun is particular or specific. The signals Definite Article: A that the noun is definite, that it refers to a particular member of a group. None vs. All None is one of the indefinite pronouns that is singular or plural. There used to be a old rule that defined that none is less than zero so mostly, it inherits a singular verb.

e.g.: e.g.: All is e.g.:

In this question I think none  none of the answers are  are correct None None  of my friends is is  coming to dinner tonight. always plural . all of my friends are all  are  coming to the party

Singular noun + of + plural noun

When "of" is preceded by a singular noun and followed by a plural one, the rule is to conjugate the following verb with the plural noun if the latter can perform the action conveyed. Otherwise, conjugate it with the singular noun preceding "of". E.g.: The swarm of flies was very dense. As you can see, the flies cannot be dense. However, the "swarm" "sw arm" is singular. Hence, "was" is singular. E.g.: He is one of the students who believe math is boring. In the above, since "students" can perform the action of believing, the verb is conjugated with the plural noun "students". This is an example similar to the original question. An exception applies in the case of "one of": E.g.: One of the students believes that math is boring. In the above, it is actually not an exception; it is about logic. Ask yourself: Do the students believe that math is boring? No. Only that one student believes so. Hence, "believes" should be third person singular. 10%, half , all, and most  most would refer to more than one thing if the object of the preposition is countable (with one possible exception that we will discuss further). Each of./ One of.that

A commonly tested GMAT concept is that of "each of/one of + plural noun". The verb that immediately follows will agree with the subject "each of/one of" if there is no relative pronoun following. E.g.: Each Each  of the witnesses was was  questioned by the police. E.g.: Each of the books reminds me of her.  Do not get bothered by it. "reminds" should still be singularly conjugated. One  of the people is... •  One Each  of the students is... Each •  Imagined a situation like this:

This website stores data such as 1% of the 100 people is/are...because, of course, 1% of 100 is one, and that's singular, right? E.g.: GMAC built 300 houses cookies to enable essential site on a large portion of the 210-acre site of the Battle of GMAC, one of only eight Revolutionary War battlefields that remain undeveloped. functionality, as well as marketing, In the above sentence, the independent clause and prepositional phrase have the same explanation explana tion as in the first example. "one of" personalization, and analytics. You will require a singular verb after. However, when there is an intruder, splitting the independent clause such as ‘ that’ , the usually may change your settings at any time relative/restrictive clause, the verb can be either singular/plural. A relative clause usually refers to the closest noun and in this case, it or accept the defaultfrom settings. is "battlefields" the prepositional phrase. Hence, ‘remain’, the immediately following verb is singular. However in the example below it is ‘rests’ and not ‘rest’. That can refer to either .one book. or .books.. The verb that follows that determines what wha t that refers to. E.g.: One of the books that rests on the table reminds me of her.

Privacy Policy

Majority of/ Percentage of

Marketing

"A high percentage of X" will be part of the exceptions in English language la nguage whereby the following verb, belonging to the same independent clause, will be conjugated singular or plural depending on the object of the preposition. Hence, Personalization "a high percentage of the population is" and "a high percentage of people are". Other such examples, but not limited to these, would include: "most of., .percent of., .Some of, .All of. and .The majority of e.g. Most Analytics of the class is not present Most of the people are gone As you can see, "most" in this case does not determine the number of the verb but the object of the preposition is (class or people). Save Accept All e.g. 10% of the students are not in the class 10% of the pie is gone

http://www.totalgadha.com

 

  I chose A, but the correct answer is B. The majority of the talk was devoted to an account of the experimental methods used by investigators in the field. a. ... b. The greater part of the talk was c. The bulk of the talk has been d. A large amount of the talk has been e. A predominance of the talk was Good one!! "majority" should be used with countable nouns only. The majority of the water is dirty. Is "unidiomatic," because "water" is a non-countable noun. Just in case, countable nouns can be counted (bottle, idea, person, brush, etc.); Non countable nouns cannot be counted (water, furniture, information, soap, luggage, etc.). There is, however, a lot of overlap between the two--beer, coke, coffee, material, love, etc. can all be either countable or noncountable, depending on the meaning, context, or level of formality. One of the most common questions is something like this: Do I say: "Most of the people is/are...?" "Most of the water is /are...?" With fractions, percentages and indefinite quantifiers, the verb agrees with the preceding noun or clause. With singular or non-count nouns or clauses, use a singular verb:   One third of this article is taken up with statistical analysis. All of the book seems relevant to this study.     Half of what he writes is undocumented.   About fifty percent of the job is routine.   All the information is current. RULE: quantifier + of + NOUN + verb

The NOUN NOUN  determines whether the verb is singular or plural. For example: Most of the people is/ are... because the quantifier "most" refers to "people," (a plural noun) so "most" is plu ral in this sentence. Most of the water is /are... because the quantifier "most" refers to "water," (a non-countable noun) so " most" is singular in this sentence. So, from these examples, you should notice that we are looking mainly at whether the object of the preposition is count or noncount because the quantifier will take on this property from the object of the preposition. In other words, in these sentences: Most  of the people are... Most "Most" becomes a count noun because "people" is a countable noun.

This website stores such Most   of thedata Most water is... as "Most" becomes a non-count noun because "water" is a non-countable noun. cookies to So, enable essential this rule tells us site only whether the quantifier is countable or non-countable. only  functionality, as well as marketing, Majority/Minority/Plurality: can be singular or plural depending on their context. When they refer to the many parts of the totality, personalization, and analytics. You they are plural. When they refer to the totality itself they are singular. may change your settings any timein GMAT Club are going to score more than 700 in the GMAT. e.g. The majority of theatmembers Thedefault studentssettings. majority is opposed to the new grade classification. or accept the The majority of students were staying in the hostel. Here "students" are acting on their own will and hence they are individual hence majority would be plural.

PrivacyPost-Exercise Policy i. I must have either chocolate ice cream _______ carrot cake to complete a great meal. Marketing ii. Because Jenny was grounded, she could neither leave the house_______ house____ ___ use the telephone. iii. When given the choice, I choose both ice cream _______ cakes. Personalization iv. The chimpanzee is much more intelligent _______ the orang-utan. v. Democrats are not so different ________ Republicans. Analytics vi. Memorizing idioms is not as fun _______ playing bingo. vii. He was so _______ he missed the______ meal. 800. viii. Scores on late the GMAT range from 200 Save Accept All ix. Many amusing quips are attributed _______ Dorothy Parker. x. Before you enter the house you are required _______ take off your hat.

http://www.totalgadha.com

 

  Answers

i. I must have either chocolate ice cream or carrot cake to complete a great meal. ii. Because Jenny was grounded, she could neither leave the house nor u se the telephone. iii. When given the choice, I choose both ice cream and cake. iv. The chimpanzee is much more intelligent than the orang-utan. v. Democrats are not so different from Republicans. vi. Memorizing idioms is not as fun as playing bingo vii. He was so late that he missed the meal. mea l. viii. Scores on the GMAT range from 200 to 800. ix. Many amusing quips are attributed to Dorothy Parker. x. Before you enter the house you are required to take off your hat. Rule 7: Quantity Words

i. 

ii. 

The words measuring quantity may be used incorrectly. incorrectly. For example, when comparing two items, it would be inappropriate to use “among” to compare them. Here’s a chart: 2 items

if more than 2

Between More Better Less

among most best least

Items that can’t be counted should not use quantity words. For example, you c can’t an’t say “fewer soup”. Here’s a chart: Non-countable words  words 

Countable words

Less fewer Amount, quantity number Much many Double: can be used with Noun/ Verb/ Adjective/ Adverb e.g.: Noun: 36 is the double of 18 Verb: The population doubled within 50 years. Adjective: An egg with a double yolk Adverb: His eyes were double bright. Twice is always adverb. Usage depends upon the structure of the sentence. In GMAT Twice as much....as, twice as many....as are frequently tested. Exercise

i. I wish there were fewer/less cars on the road. ii. The amount/number of students in my classes has been increasing lately. iii. Between/Among the four of us, I am the tallest. iv. Of the entire class, he is the taller/tallest. Answers

i. ’Fewer’ is correct here because cars are countable. No ’Less’ does not work here because cars are not countable ii. ’Amount’ is not correct because ’students’ are countable ’Number’ is correct because students are countable iii. ’Between’ is used only when two th ings are being compared. ’Among’ is used when more than two things are being compared iv. You only use the comparative (-er) when you’re comparing two things.You use the superlative (-est) when you’re comparing three

This website such as (-est) form when you are comparing three or more things, the comparative (-er) when you compare or morestores You usedata the superlative two cookies to enable essential site functionality, as well as marketing, Rule 8:-Idioms personalization, and analytics. You that are common in standard written English. There is no magic formula or logical procedure for Idiomatic expressions are phrases determining a particular combination of verbs, prepositions, and nouns is correct – you simply have learned these expressions may change your ifsettings at any time beforehand, or you have not. Here is an example of an idiomatic expression : I forbid you to go. or accept the default settings. Why forbid to? Why not forbid from? Or forbid with? It is simply forbid to and not something else because centuries of custom and use have made it that way. We must 1) accept that, and 2) learn that. Watch for the prepositions (to, the, of , at , for , on, in, about , etc.,) changing among the answer choices. This usually Privacy1.Policy implies an Idiomatic problem, if not a Parallel Construction problem.

Marketing 2. Note that Idiomatic problems are often shorter than many of the other types of questions. Not in all cases, but as a general rule, if the problem seems much shorter than other problems, it is probably Idiomatic. Personalization Analytics

3. Check out the verb immediately before the changing preposition. Which verb-preposition combination sounds worst? Eliminate answer choices that sound just awful, ex. He forbids me of going . The correct answer would be He forbids me to go. Save Accept All 4. Which verb-preposition combination sounds best? Choose that as your answer.

http://www.totalgadha.com

 

  Example:

The A B C D E

Duke of Argyle told us that we could count with him to be there on time. with him to be there with him on being there on him to be there on him for being there to him to be there

Explanation: 

The correct answer is C. We know it is an Idiomatic I diomatic problem because a) the prepositions change among the answer choices (with, with, on, can on, and to), and 2) the problem is shorter than the We eliminate sounding unidiomatic. So average eliminateSentence E. CountCorrection better than count with, so eliminate A and B. count to for on seems problem. Between C and D , count on him to be ther there e sounds better than count on him for being there , so choose C as your correct answer. Major Idioms you should be pretty familiar with: 1)  ability to verb; ability of noun 2)  a debate over 3)  a lot 4)  a responsibility to 5)  a result of 6)  a sequence of 7)  acclaimed as is the correct idiom (Acclaimed to be is wrong) 8)  accompanied by.... 9)  adapted for 10)  Adverb twice cannot be an object of proposition ‘by’. ‘Increase by twice’ is incorrect; ‘doubled’ is correct 11)  affect to.. 12)  agree with 13)  Aid in (Aid for is incorrect) 14)  Allergy to (Allergy of, allergy for are incorrect) 15)  Allocated to is the correct idiom 16)  alternative to.... 17)  as a result of... 18)  as an instance of 19)  as good as...or better than 20)  as great as 21)  as much as 22)  Associate X with Y 23)  assume ...to be of... 24)  At least as strong as(At least as great as) 25)  Attempt to ‘do something’ (Attempt at doing is incorrect). 26)  attend to (someone) 27)  attribute X to Y/X is attributed to Y 28)  based on 29)  believe X to be Y 30)  Believed to have 31)  benefit from... 32)  better served by X than Y .. 33)  between X and Y 34)  Both X and Y (Both X as well as Y is incorrect) Both at X and at Y is correct. Both on X or on Y is correct. This website data such as- Is a singular word   35)stores Business ethics   call...to consider... cookies to 36) enable essential site 37)  centres on functionality, as well as marketing, 38)  Combined X with Y OR Combined X and Y (Both are correct) personalization, and analytics.forYou 39)  Compensate 40)your   Concerned - worried; may change settings atfor any time concerned with - related/affiliated 41)  conform to or accept the default settings. 42)  Consider X to be Y (a little controversial) 43)  contrary to... 44)  created with 45)  Credit X Rupees to Y’s account (When money is involved) Privacy Policy 46)  Credit X with discovering Y (Credit with doing something) 47)  decline in.... Marketing 48)  deciding that 49)  defined as 50)  depends on whether Personalization 51)  depicted as 52)  Descendent of (Descendent for is incorrect) Analytics 53)  Different from one another (Different one from the other is wrong) 54)  Distinguish between X and Y (2 very different items, distinguished, say red and green colors) 55)  Distinguish between Save Accept All X and Y (Distinguish X from Y is incorrect) 56)  Distinguish X from Y (Two pretty similar items, say original paintings from fake ones) 57)  doubt that

http://www.totalgadha.com

 

  58)  59)  60)  61)  62)  63)  64)  65)  66)  67)  68)  69) 

either...or enable to entrusted with... Estimated to be (Estimated at is incorrect) expected that X would be Y ... expected X to be Y ... extent to ... fascinated by for jobs.. for over...XXX years... forbid X to do Y identical with X forbidding Y from doing Z

70) forcing 71)   From X ...to... to Y (Grow from 2 million to 3 billion) (From X up to Y is wrong) 72)  Given credit for being ones - who 73)  had better(do) 74)  In an attempt to (gain control) 75)  in contrast to 76)  independent from 77)  indifferent towards 78)  Intent on 79)  interaction of ... 80)  Just as - So too 81)  May be (This is a word) is idiomatic, maybe (This means perhaps) is not idiomatic 82)  Mistake X for Y 83)  Making do 84)  modelled after 85)  merge X with Y 86)  more than ever 87)  more X than Y ... 88)  more...than / less...than 89)  more...than ever... 90)  must have (done) 91)  Native of (Native to is also used in some cases) 92)  Neither - Nor should have parallel forms associated to it. 93)  no less....than 94)  No sooner than 95)  Not in a flash but in a 96)  not only...but also 97)  Not so much to X as to Y 98)  not X ...but rather Y .. 99)  noted that .. 100)  one attributes X (an effect) to Y (a cause) 101)  One X for every ZZ( some numeric number) Y's ... 102)  Persuaded X to do Y 103)  Plead guilty for failing 104)  Potential for causing 105)  potential to 106)  prohibits X from doing Y 107)  range from X to Y 108)  range of ... This website stores data such   109) reason….. thatas incorrectly seen as reason….. because    “Regard as”site is the correct idiom -- Regarded Regarded as having, Regarded as ones who have cookies to 110) enable essential 111)  regardless functionality, as well as marketing, 112)  regards X as Y ... personalization, analytics. You 113)  and replacing with... 114)   Require that X betime Y (Not require that X is Y) may change your settings at any 115)  research to or accept the default settings. 116)  responsible for 117)  restitution...for ... 118)  resulting in 119)  retroactive to Privacy Policy 120)  resemblance between X and Y 121)  Same as X..as to Y Marketing 122)  same to X as to Y 123)  seem...to...(seem is plural) 124)  so (adjective) that Personalization 125)  So X as to be Y (So unreal as to be true) 126)  So X that Y (So poor that they steal) Analytics 127)  subscribe to 128)  such...as 129)  targeted at Save Accept All 130)  that X ...that Y ... 131)  That X is called for is indicated both by Y and by Z.

http://www.totalgadha.com

 

  132)  133)  134)  135)  136)  137)  138)  139)  140)  141)  142)  143) 

the same to X as to Y to .. used to (example to get used to or to become used to) to contrast X with Y To exchange X for Y (exchange X with Y or any other form is incorrect) to mistake X for Y to monitor ... to orbit... To ratify (At ratifying is incorrect) An attempt to ratify is the correct use to result in to sacrifice X for Y to survive To try to fix is the right idiom (to try and fix is incorrect)

144) 145)   146)  147)  148)  149)  150)  151)  152)  153)  154)  155)  156)  157)  158)  159)  160)  161)  162)  163) 

To worry about someone’s condition (To keep worrying over an action) used in the construction... used to (do) valid for viewed marriage as way to provide (Way for providing is incorrect) When ‘rates’ means ‘prices charged’ it should be followed with ‘for’ widely anticipated that.... Worried about (When talking about someone’s condition) X [is] expected to Y X as Y .. X forbids Y to do Z ... X is attributed to Y X is different from Y (different than Y is incorrect) X is to what Y is to X is unknown, nor it is known - is a correct idiom (Neither is not required) X ordered that Y be Z'ed... X ordered Y to be Z'ed.. X ordered Y to do Z X prohobits Y from doing Z ...

Context based idioms to sympathize .. noun; for sympathizing sympathizing  ...verb

e.g.: to sympathize the regime (noun) e.g: for sympathizing with the regime (verb) SC : 1000-190 from’ is used when result is used as a verb ‘result from’  eg. Bad temper results from lack of sleep ‘result of’  of’ is used when result is used as a noun eg. Bad temper is a result of  lack of sleep  ‘Acclaim as’ and  and ‘  ‘acclaim to be’ both are right. But it depends on the sentence which one to use ‘Aim for’ for’  is used with Noun/ Gerund. e.g.: I wan’t to aim for a better school. ‘Aiming to’ to’  is used with Verbs. e.g.: I am aiming to score well in the GMAT. Refer : SC-1000-106 ‘Aid in’ + doing something e.g.: The UN’s aid in alleviating poverty in Africa is not enough. e.g.: The UN’s aid to African countries is not enough. Exercise

This website stores data such as i. She is not only beautiful, cookies to enable essential site________ smart. ii. I can’t distinguish day _______ night. functionality, as well as marketing, iii. I can distinguish between black _______ white. personalization, analytics. You iv. You haveand a responsibility ____ take care of the child. v. You are responsible the child. may change your settings ______ at any time vi. Art historians regard the Mona Lisa _______ one of the or accept the default settings.

greatest works of art. vii. Art historians consider the Mona Lisa _______ one of the greatest works of art. viii. He is not so much smart _______ cunning. ix. My GMAT teacher defines the conclusion _______ the main point of the argument. Think of it more _____ a promise than _____ threat. Privacyx.Policy xi. Many people see euthanasia _________ an escape from pain. xii. The hypothesis ________ aspartame causes brain tumors has not been proven definitively yet. Marketing xiii. Mel Gibson is a native _________ Australia. xiv. The kangaroo is native ________ Australia. Personalization xv. My pasta sauce is far superior __________ my mother-in-law’s. xvi. Try _______ stay awake during the essay section of the test.

Analytics

Answers

i. She is not only beautiful, but also smart. If you have not only you must have but also as well ii. I can’t distinguish day from Save Accept Allnight. You distinguish something from something else iii. I can distinguish between black and white. distinguish between . . . and . . . iv. You have a responsibility to take care of the child.You have a responsibility to do something

http://www.totalgadha.com

 

  v. You are responsible for the child. You have a responsibility for something/someone vi. Art historians regard the Mona Lisa as one of the greatest works of art. Regard as is the correct idiom here vii. Art historians consider the Mona Lisa one of the greatest works of art. Nothing goes in the blank in this case. ca se. The word consider needs no other words for a correct idiomatic expression viii. He is not so much smart as  as cunning. Not so . . . as is the idiom to remember here ix. My GMAT teacher defines the conclusion as  as the main point of the argument. Define as . . .is the idiom here. x. as . . . a xi. as xii. that xiii. of xiv. to xv. to xvi. to There are three types of idioms that you'll see on the GMAT: 1.  word pairs that go together 2.  prepositions and the verbs that use them 3.  standard expressions. Word Pairs

as…as

The movie was as long as it was boring.

more…than less…than greater….than

The workshop was more thrilling than anything I'd ever done.

not only…but (also)

She was not only exhausted but (also) famished as well.

so…that

The apartment was so expensive that no self-supporting student could afford it.

(just) as…so neither…nor either…or

(Just) as it is the duty of employees to contribute to the well-being of the company, so it

is the duty of the company to contribute to the well-being of its employees. Neither a borrower nor a lender be.

IDIOMATIC PREPOSITION USAGE

Many idiomatic expressions tested on the exam involve prepositions. There's no overarching grammatical rule that tells you which prepositions go with which verbs. Again, the rules are determined by usage, so you'll have to ‘listen’ to the expression and determine if the verb is followed by the correct preposition. Be on the lookout for commonly tested prepositions like ‘of’, ‘at’, ‘by’, ‘in’, ‘from’, ‘to’, and ‘for’. If you have difficulty determining whether a usage is correct, try testing out the idiom in a simpler version of the sentence. Example: Although he was considered as a leading proponent for the controversial new initiative, the professor nevertheless sought refuge from the media uproar.

The sentence above becomes: The professor was considered as a proponent. Does anything sound unusual? Could this sentence be worded differently? In fact, to be, not as, is the correct idiom:

This website stores data such as to be  a proponent. The professor was considered cookies to enable essential site Would rather rather   present tense if referring to myself and past tense if someone other than the subject is doing the action functionality, as well asWould marketing, Example: rather I speak  present tense Example: Would rather personalization, and analytics. You you spoke  past tense may change your settings at any time General Grammar definitions or accept the default settings. So = therefore So that = in order to/in order that That = the fact that

Privacy Policy Conjunctions that can join two independent clauses are: and, but, yet, for, or, nor Don’t start sentences with “Because”

Marketing

Words requiring ‘how’  requiring ‘how’  (example:  (example: know how) Know Personalization Teach Learn Analytics Show Always choose active over passive voice

Save Accept All Active example: Elaine purchased new software for the company Passive example: New software was purchased for the company by Elaine

http://www.totalgadha.com

 

  Another frequently tested grammatical error is unnecessary use of the passive voice. It's a good idea to become familiar with this type of error; it appears quite often in the answer choices of sentence correction questions. As a reminder, the passive voice is in use when the action of the sentence is performed on the subject. The active voice is in use when the subject itself performs the action. Example: Active: Allison (subject) went (action) to the store to buy a cake (object). Passive: The cake (subject) was bought (action) by Allison (object). Preposition use  “To” or “Of” needs subject + verb NOT object + verb Examples she,him, he, whoever, who, Examples of of subject: object: her, whomever, its, Iit Either side of a form “to be” (were, was) must have subjects not objects on either side to agree. A verb that ends in –ing is a gerund gerund which is a verb acting like a noun. Try not to use gerunds if you can help it. Adverbs, not adjectives modify verbs

Key identifier: Adverbs end in –ly Correct example: I sure wish I were rich Incorrect example: I surely wish I were rich Word Usage

Some pairs of words, like fewer and less, are often used incorrectly because they're treated as synonyms. In fact, there is a solid rule that determines which one you should use, and the exam will test your ability to decide which is the correct option. The exam tests four such word pairs with particular frequency, so memorize the rules pertaining to them if you don't know them already. 1.  Fewer/Less  Which of the two following sentences is correct? 

  

The company company fired fired no no fewer less than fifty employees. The than fifty employees. The second sentence is correct. Why? Because you use less when you're talking about things you can't count (less pollution, less violence) but fewer when you're talking about things you can count (fewer pollutants, fewer violent acts).

2.  Number/Amount 

These words follow the same rule as less and fewer. Number is correct when you can count the thing being described (a number of cars, a number of people) and amount is correct when you cannot (amount of love, amount of pain). 3.  Among/Between 

Use between when only two options are available (between the red car and the blue car) and among when more than two options are available (among the five answer choices, among the many books). 4.  Compared to v/s. Compared with:  

To show comparison between unlike things, ‘compare to’ is used. To show comparison between like things,  ‘compare with’ is used. This website stores data such as e.g. cookies to enable essential site her to a summer day.   He compared   Scientists compare the human brain brain to a computer. computer. (Unlike thing)  functionality, as well as marketing,   The police compared the forged signature with the original. (Like things) personalization, and analytics. You which you should consider. There are two rules

may change your settings at any time Compare usually takes the preposition to when it refers to the activity of describing the resemblances between unlike or accept the default settings. things:

He compared compared  her to  to a summer day. • Scientists sometimes compare compare  the human brain to to  a computer.

• 

Privacy Policy Marketing

Compare takes with when it refers to the act of examining two like things in order to discern their similarities or differences:

Personalization with the original. • The police compared the forged signature with  Analytics Save

• The committee will have to compare compare  the Senate's version of the bill with  with the version that was passed by the House. compare  is used to mean ‘to liken’  liken’ (one) with  with another, with with  is traditionally held to be the correct preposition: When compare That little bauble is not to be compared with (not to) this enormous jewel. But ‘to’ is frequently used in this

Accept context and isAll not incorrect.

Rule 1: Compare to  to compares unlike things, whereas compare with with  compares like things.

http://www.totalgadha.com

 

  Rule 2: Compare to  to is used to stress the resemblance. Compare with  with can be used to show either similarity or difference but is usually used to stress the difference . There is a difference between compare to and compare with; the first is to liken one thing to another; the second is to note the resemblances and differences between two things. 5.  Each

This question tests one use of "each" which most of us ignore. The traditional traditiona l rule still holds true i.e. ‘the subject of a sentence beginning with each is grammatically singular".

But there is another rule which says that: When each follows a plural subject, the verb and subsequent pronouns remain in the plural :

e.g. the apartments each have their own private entrances (not has its own private entrance)    

Three cats each eat ... Three cats, each of which eats ...,

In 1, each is postpositive Adj, whereas in 2, it is distributive determiner. e.g.  

Television can be superficial, as when three major networks each broadcast exactly the same.

Adverb clause of manner with temporal adverb clause: Television can be superficial, as [TV [TV is superficial] superficial] when three networks each broad cast the same. -----------------------------------------------------------------------------------------------------------quote:  

Although it claims to delve into political issues, television can be superficial such as when each of the three major networks broadcast exactly the same statement from a political candidate. (A) as sometimes when each occur of the ifthree networks (B) superficial superficial,such as can all ofmajor the three major networks (C) superficial if the three major networks all (D) superficial whenever each of the three major networks (E) superficial, as when the three major networks each First of all, each, each , if it's a pronoun (as it is in A), is singular. In fact, each is almost always singular, but there's at least one exception. So, A can be faulted for using a plural verb, broadcast , with a singular subject, each. Option A uses such as, which is used to give examples. All the other incorrect answer choices use words that mean something different from for example. The best answer, E, maintains the same meaning as A, and corrects the subject/verb agreement problem. Please note that one of the accepted meanings of as is for instance, and with this meaning, as is an adverb and can therefore be followed by parts of speech other than simply nouns. B is not only awkward, it also incorrectly uses if in the subordinate clause connected with can in the main clause. I think this is the part that is confusing , so let's flip the sentence around to see a bit more clearly that it's not correct to use if with can: If all of the three major networks broadcast the same statemen statement, t, television can be superficial. This sentence should read: This website stores data such as If all of the three major networks broadcast the same statement, statement, television will be superficial. cookies to enable In essential site it's not correct to use can after if (in the context of what we've been talking about). Let's look at a simpler other words, example: functionality, as well as marketing, If the temperature drops below 0 degrees celsius, distilled water can or will freeze. personalization, and analytics. You It is better to use each than it is to use all , since each network is operating independently-- all implies that the networks may change your settings at any time were working together. or accept the default settings. 6.  The use of being 

People who study for GMAT for a while quickly learn that being is usually wrong.

Privacy Policy Marketing

So I'm guessing you already know that being in an answer choice is wrong more often than it is right. This is a good strategy to get you started, but to get over 700 on the GMAT, you really need to know some of the finer points of GMAT Sentence Correction that relate to the use of the word being. There are at least two different situations in which being is often the right answer.

Personalization Analytics Save

Here is the first example of when being is correct:  a. 

When the grammar requires it. it .

Many ideas can be expressed in more than one way. For example, I can say:

Accept All

I'm afraid of being late. late.  I'm afraid that I'll be late. late.  

http://www.totalgadha.com

 

  Each has its own emphasis, but the point is that these two structures exist. Whether we can express ideas in one or more structures is really related rela ted to the word used; in other words, it is idiomatic. But some idioms allow only one structure. For example: In addition to being one of the first restaurants to combine Mediterranean and American tastes, Chez Panisse in Berkeley is also one of the Bay Area's most established restaurants. The idiomatic structure in addition to does not have a counterpart that uses a subject and a verb, so our only option here is to use being, which is grammatically a noun, but is derived from a verb.

w i t h + NOUN + b e i n g + NOUN COMPLEMENT

b. 

The second example of when being is correct is shown in this example: There are many reasons to get an MBA, with increased career prospects being the most important for many MBA applicants. Technically this part here: with increased career prospects being the most important for many MBA applicants

is an absolute phrase, but I think it's also h elpful just to memorize the pattern: Framed by traitorous colleagues, Alfred Dreyfus was imprisoned for twelve years before there was exoneration and his freedom. (A) there was exoneration and his freedom (B) he was to be exonerated with freedom (C) exonerated and freed (D) being exoneration and his freedom (E) being freed, having been exonerated C is correct. The main thing here is that we do not have the best option available to us:  

before he was exonerated and freed The second best option would be:

 

before being exonerated and freed

7.  Because v/s. In That:  That:  When ETS puts ‘because’ and ‘in that’ in a sentence, more often than not, ‘in that’ would be correct. ‘In that qualifies’ the previous sentence, while ‘because’ is just used to show a simple causal relationship.  

Teratomas are unusual forms of cancer because they are composed of tissues such as tooth and bone not normally found in the organ in which the tumor appears. because they are composed of tissues such as tooth and bone because they are composed of tissues like tooth and bone that are This website stores data such as because they are composed of tissues, like tooth and bone, tissues •  cookies to enable •essential   in that site their composition , tissues such as tooth and bone, is inmarketing, that they are composed of tissues such as tooth and bone, tissues •  as functionality, as well •  • 

personalization, and analytics. You E is the correct answer.  may change your settings at any time 8.  Usual v/s. Is Usual:  Usual: He is faster than is usual for any human being – Is correct. or accept the default settings. He is faster than usual today – is correct

A Mercedes is more expensive than usual for a car – Incorrect

Privacy Policy

A Mercedes is more expensive than is usual for a car – Correct

Marketing

When something is compared to a subgroup to which it belongs, is usual should be used. When something is compared to itself, usual is fine.

e.g. He is nicer than usual. Personalization Could: If you are just assuming something, ‘could’ should be used. 9.  Can v/s. Could: 

Analytics Save

e.g. Artificial intelligence emerged during the late 1950's as an academic discipline based on the assumption that computers are able to be programmed to think like people.

Accept All

(A) are able to be programmed to think like people

http://www.totalgadha.com

 

  (B) were able to be programmed to think as people (C) can be programmed to think as people can (D) could be programmed to think like people (E) are capable of being programmed to think like people do Could is used for: possibility (John could be the one who stole the money), condition (If I had more time, I could travel around the world), suggestion (You could spend your vacation here), polite request (Could I have something to drink?) 10.  Like v/s As 

GMAT grammar rule mentioned here will have some exceptions, for the reason that we are discussing GMAT patterns here and not "English grammar rules". It is very difficult to give a pattern that applies in every case. I would say that generally speaking, your summary is good, but just to be sure, I want to restate: Use like when you want to focus on two nouns; Use as when you want to focus on two nouns doing two actions. Another little trick is that "just as" can replace "in the same way that..." Let's compare two very similar sentences that could cause confusion: My Siamese cat moved across the floor just like a lion stalking its prey. To me, this sentence stresses how two different cats are similar. T his is confusing because we have a noun, "lion" and a participle "stalking," which would seem to indicate that we should use "as," but it's just not so. In this sentence, do you think we're trying to say that  a lion stalks its prey. My Siamese cat moved across the floor in the way that

I don't think so... My Siamese cat moved across the floor just as  as a lion stalks its prey. how  the cat is moving. Furthermore, at some point, we are going to This one sounds bad because we are not explaining how run into some ambiguity--"as" does also mean "at the same time," and I also think that the sentence above does sound a bit like two things are happening at the same time.

Let's look at a better sentence: as  a lion stalking its prey moves. My Siamese cat moved across the floor just as This one sounds very good; it explains how my cat moved. Furthermore, it has the same meaning as: My Siamese cat moved across the floor in the way that  that a lion stalking its prey moves.

11.  Not/But vs. Rather than

The key here is to realize that not... but... is conjunction. We use conjunctions when we want to join things that are "linguistically equivalent." Help much? No, probably not. How about some examples? Pucci is not a d og  but a ca t .   Not Todd  but T a k a  a   will be studying with us today.   I not was s ad but h a ppy  to learn that Megumi was moving to Paris for a better job.  

This website stores data such as should notice that the words in bold are "linguistically equivalent," or, as we say in class, "parallel." Now compare cookies to enable You essential site one of these sentences if you try to use rather than: functionality, as well as marketing,   analytics. Pucci is a cat personalization, and Your a t h e r t h a n  a dog. may change your settings at any time Doesn't this sentence sound crazy? It should; the meaning is all wrong. Now, let's look at a similar sentence, one in which or accept the default settings. rather than is okay:  

Privacy Policy

I want a cat r a t h e r t h a n  a dog.

This sentence is okay because we are expressing a preference  preference for one thing over another thing.

Marketing

I need X, not Y = I need X but not Y = I need not Y but X "I need X rather than Y" does not connote "I need not Y", it just tells your preference.

Personalization

12.  Use of Consider: When ‘consider’ means ‘regard as’, ‘as’ should not be present with ‘consider’ in the sentence. Consider

Analytics Save

must directly be followed by the sentence without an infinitive like ‘to be’ etc. Example: Some students of literary criticism consider the theories of Blaine to be a huge advance in modern critical thinking and question the need to study the discounted theories of Rauthe and Wilson.    

Accept All

to be a huge advance in modern critical thinking and question as a huge advance in modern critical thinking and question

http://www.totalgadha.com

 

   

as being a huge advance in modern critical thinking and questioned

 

a huge advance in critical thinking and question (When consider means regard as, no need of as)

 

are a huge advance in modern critical thinking and questioned

Example 2: Critics consider correction facilities to be an integral part of  communal system. - Incorrect Critics consider correction facilities an integral part of  communal  communal system. – Correct OF:  On GMAT, ‘Because’ is preferred over ‘on account of’. This is because ‘because’ can 13.  BECAUSE v/s. ON ACCOUNT OF: 

introduce an entire subordinate clause in the sentence (Golden crab is not fished, on ac count of living… - is not correct). Golden crab is not fished, because it lives… - is correct 14.  CHIEF of which v/s CHIEF among which :

Jamieson's proposal was rejected for several reasons, the chief among which was cost. (A) the chief among which was cost (B) among which the chief was its cost (C) the main one was cost (D) the chief reason of which was its cost (E) the chief of which was cost chief  means  means main or principal , and all three words mean the top one. And when we refer to a member of a group, we use of . For example, if we want to name our best friend, we'd say He is the best of my friends, but not He is the best among my friends.

Now, run these through your head to see how they sound: The main reason of all the reasons. The main reason among all the reasons.   I think the first sounds better and is more precise.

 

PRACTISE:  15.  PRACTICE v/s. PRACTISE: 

Practise is a verb and practice is a noun. I practise piano is correct. Also, I had my piano practice for the day is correct. The doctor practiced for 20 years; his brother a lawyer had a 2 year practice. 16.  EACH v/s EVERY:

Each refers to ‘2 times times’’, every refers to ‘More than 2 times’ times’  The dog has bitten my younger son twice, and each time, he has had to be sent to his kennel. each time, he has had to be sent to his every time, it has had to be sent to his each time, it has had to be sent to its

every time, it has had to be sent to its each time, he has had to be sent to its  ‘Each time’ because it has bitten only twice. The use of the pronoun pronoun "he" is unclear here - does it refer to the dog or to

This website stores data such as the son? Inword fact, tracing our way back along the sentence, we find that we come across the word "son" before we come the cookies to enable across essential site "dog", which rather implies that it is the son who owns, and is sent to, the kennel. The way to get round this ambiguity is to use "it" followed by "its" (no apostrophe!). This limits the choices to (C) and functionality, as well asThe marketing, (D). only difference between these options is the fact that one uses the word "every" and the other uses the word "each". Since these personalization, and analytics. You refer to the two times that the dog bit the son, we shouldn't use ‘every’ (that refers to three or more times). The correct option is therefore (C). may change your settings at any time or accept the default settings. 17.  Economic v/s. Economical: Economic means "having to do with the economy or the study of economics." Economical means "careful or prudent in managing finances, money- saving."

Privacy Policy

The supply of oil being finite has become an economical and political consideration of the first magnitude for all modern industrial nations. (A) The supply of oil being finite has become an economical (B) The finite supply of oil has become an economical Personalization (C) That the supply of oil is finite has become an economical The supply of oil being finite has become an economic Analytics (D) (E) That the supply of oil is finite has become an economic  economic 

Marketing

Save

18.  Due To

Accept All

 ‘Due to’ means  means ‘  ‘caused by’ It should only be used if it can be substituted with ‘ caused by by’’. It does not mean the

http://www.totalgadha.com

 

  same thing as "because of." Incorrect: The game was postponed due to rain. Correct: The game was postponed because of rain. Correct: The game's postponement was due to rain. 19.  Neither … Nor Neither  the prosecutor’s eloquent closing argument nor Neither nor  the mountains mountains  of incriminating evidence were were  able to convince

the jury to find the defendant guilty. nor  sentences, the verb has to agree with the subject following nor nor  - in this case mountains, which is In neither … nor plural. or sentences, the verb must agree with the subject following or . When you see .. neither .. or .. Likewise in either .. or  nor in a sentence, see if it fits this sequence nor  Neither (A or B), nor C !!! also, not (A or B), nor C is fine too.

20.  So [adjective] as to [verb]

Correct: Her debts are so extreme as to threaten the future of the company  “So as” is never correct on the GMAT Incorrect: He exercises everyday so as to build his stamina Correct: He exercises everyday in an effort to build his stamina

21.  None/No one  None can be singular or plural.  No one is always singular. 22. Greater than/More than  ‘Greater than’ is appropriate when describing numbers alone. e.g.: Greater than 100…’

‘More than’ should be used when describing the numbers of objects or when making comparisons. e.g.: More than 100 fish. 23. Thinking words (i.e., theory, belief, believe…) + that Thinking words are always followed by that “Lucy’s belief that the Earth is flat is great.” (Correct) “Lucy’s belief of…” (Incorrect)

Not To Use:

English grammar not only has rules, but it also has preferences. • There are certain ways of saying things that, although not technically wrong, are still considered ’not preferred.’ • We call these constructions non-fatal errors.

This website stores data such as • Non-fatal errors come into play when comparing the remaining answer choices after eliminating elimina ting those with standard errors. site cookies grammatical to enable essential • Sometimes youas will have more than one answer choice that contain no grammatical error and preserve the meaning of the functionality, as well marketing, original sentence. personalization, and analytics. • In these situations, chooseYou the answer with the preferred construction. • If more than one answer choice contains no grammatical errors and no changes in meaning, non-fatal errors will help you may change your settings at any time make the right choice. or accept the default settings. 1)  Any Sentence construction with ‘ preposition + noun + participle ’ e.g. With child-care facilities included (with – preposition, child-care – noun, included – participle)

Privacy Policy

Infinitives like ‘to include’, ‘to implement’ etc. are wrong on GMAT. Instead use, ‘implementing’, ‘including’ (Which are known as Gerunds) etc.

Marketing

A Gerund is a noun formed from a verb i.e. overcoming the GMAT is a great achievement. Infinitives are a re usually formed to retain the integrity of the idiom which is used or to keep a sentence in parallel Personalization 2)  Analytics

Hopefully, is almost always wrong.

Save

Charlotte Perkins Gilman, a late nineteenth-century feminist, called for urban apartment houses including child-care facilities and clustered suburban houses including communal eating and social facilities. Accept All A) including child-care facilities and clustered suburban houses including communal eating and social facilities B) that included child-care facilities, and for clustered suburban houses to include communal eating and social facilities

http://www.totalgadha.com

 

  C) with child-care facilities included and for clustered suburban houses to include communal eating and social facilities D) that included child-care facilities and for clustered suburban houses with communal eating and social facilities E) to include child-care facilities and for clustered suburban houses with communal eating and social facilities included

First, we can eliminate all the answer choices that use with communal eating and social facilit ies included or any other structure that follows this pattern: preposition + noun + participle This pattern is almost always wrong on the GMAT, and is certainly wrong in this question. This eliminates C) and E). A) is ambiguous--is clustered suburban houses parallel with child-care facilities or with urban apartment houses? We need to repeat for to be sure that clustered suburban houses is parallel with urban apartment houses. Now we're left with B) and D). Now, the only difference between B and D is the infinitive to include, wh ich GMAT considers unidiomatic. That leaves us with D).

Few Important Points to remember:

1) (less preferred) being < since < because  2) As Such such = nominal equivalent to the foregoing clause.

Clause1 and as such, Clause2 = clause1, and as clause1, clause2. Caesarea was Herod’s city, founded as a Romanized counterweight to Hebraic Jerusalem, and as such such  it was regarded with loathing by the devout. The meaning of the above sentence is similar to the following: Because Caesarea was Herod’s city, founded as a Romanized counterweight to Hebraic Jerusalem, it was regarded with loathing by the devout.

3) Reduced Costs = Reduction IN costs (result of reduction)  ‘Reduction of’ is used when reducing by a certain amount. e.g. reduction of 20%.

Second, f or al l means d e s p i t e, Second, f e , and a lo lo n g w i t h means i n a ddi t i o n t o . I'm sure you'll agree that the meanings are different, right? For example: I haven't visited Bora Bora, and neither has Kerry [visited Bora Bora].In this case, I can omit visited Bora Bora because it already appears in the sentence. Let's look at another example: I haven't visited Bora Bora, and I probably never will visit Bora Bora. This is wrong, at least on the GMAT, since visited and visit are different.

4) It quote:

This website stores data such as 1) Why the answer is E? at I chose A determined cookies to Schliemann enable essential site the age of seven to find the site of ancient Troy and devoted his subsequent career to do it. a)... functionality, as well as marketing, b) has devoted his subsequent career to do that c) devoted his subsequent personalization, and analytics. Youcareer to such an end d) has devoted his subsequent career for that may change your settings at any time e) devoted his subsequent career to that end or accept the default settings. Option A here is wrong since it uses the pronoun ‘it’ replaces ‘find the site of ancient’ ANTECEDENT.  First off, every single time you see a pronoun, especially the word ‘it’, you MUST CHECK THE ANTECEDENT. 

Privacy Policy This question is a favourite one--using ‘it’ to replace a sentence. In GMATland, ‘it’ must always replace a noun. For example, this sentence would be wrong in GMATland:

My little brother said I took his cookies, but I didn't do it."it" doesn't replace any noun; it " tries" to replace a sentence: ‘I took his Marketing cookies’. The correct phrase is ‘  ‘helpful in demonstrating’  demonstrating’ and not ‘  ‘help help to demonstrate’. demonstrate’. Personalization  ‘until’ is used to express a point of time in the future. 5) Analytics as example, long as’ implies  ‘for one thingaswill occur while another 6) ‘we will that stay outside long as it's light out.’ thing is still true; Save Accept All

7) Semicolon: Any sentence after a semicolon (;), should be an independent clause.

http://www.totalgadha.com

 

  the: The pattern to remember is one of the NOUN (this noun will always be plural) + that/who + Plural verb 8) One of the:  Example ::  He is one of the persons who make money. This is one of the cars that run on hydrogen. A number of people are waiting for the bus. The number  number of cars in the city is decreasing.

Any of the stockholders who disapprove – is the right use. Three cats, each eat. Three cats, each of which eats.

9) Resumptive modifiers Since the 1930’s aircraft manufacturers have tried to build airplanes with frictionless wings, shaped so smoothly and perfectly that the air passing over them would not become turbulent.  

wings, shaped so smoothly and perfectly

 

wings, wings so smooth and so perfectly shaped

wings that are shaped so smooth and perfect wings, shaped in such a smooth and perfect manner wings, wings having been shaped smoothly and perfectly so   B is the correct answer.

 

 

Wings is required to unambiguously convey the meaning of the sentence. A resumptive modifier picks up a word or phrase from a sentence that seems to be finished and then adds information and takes the reader into new territory of thought. In a crowded, acquisitive world, the disappearance of lifestyles such as those once followed by southern Africa's Bushmen and Australia's aborigines, requiring vast wild spaces and permitting little accumulation of goods, seem inevitably doomed.          

requiring vast wild spaces and permitting little accumulation of goods, seem inevitably doomed requiring vast wild spaces and permitting little accumulation of goods, seems to be inevitably doomed which require vast wild spaces and permit little accumulation of goods, seems to be inevitably doomed life-styles that require vast wild spaces and permit little accumulation of goods, seem inevitable life-styles requiring vast wild spaces and permitting little accumulation of goods, seems inevitable

E is the correct answer. • 

The Swiss watchmakers' failure to capitalize on the invention of the digital timepiece was both astonishing and alarming — astonishing in that the Swiss had, since the beginnings of the industrial revolution in Europe, been among the first to capitalize on technical innovations, alarming in that a tremendous industrial potential had been lost to their chief competitors, the watchmakers of Japan.

• 

The defensive coaches taught risk-taking, ball-hawking, and perpetual movement — three strategies that bewildered the opposition and resulted in many bad passes, steals, and easy fast break baskets.

Another example in the same league… Proponents of artificial intelligence say they will be able to make computers that can understand English and other human

This website stores data such as languages, recognize objects, and reason as anpurposes expert does—computers deciding whether to authorize a loan, or other such as these. that will be used to diagnose equipment breakdowns, cookies to enable essential site an expert does—computers that will be used to diagnose equipment breakdowns, deciding whether to authorize a loan, or functionality,(A) asaswell as marketing, other purposes such as these personalization, and analytics. You (B) as an expert does, which may be used for purposes such as diagnosing equipment breakdowns or deciding whether to may change your settings at any time authorize a loan or accept the default (C) like ansettings. expert—computers that will be used for such purposes as diagnosing equipment breakdowns or deciding whether to authorize a loan

(D) like an expert, the use of which would be for purposes like the diagnosis of equipment breakdowns or the decision whether or not a loan should be authorized Privacy Policy (E) like an expert, to be used to diagnose equipment breakdowns, deciding whether to authorize a loan or not, or the like C is the answer. Marketing e.g. 1)  1) His father demanded that he return home by 9 PM.

Personalization Return – Simple present tense DemandedDemanded- precedes ‘that’ in the sentence. Analytics

i.The manager demanded that the staff stay late to finish the work. Incorrect (Should  (Should must be removed) ii.Jack suggested that Sylvia should buy a new watch – Incorrect Save Allbuy a new watch. Jack suggestedAccept that Sylvia

http://www.totalgadha.com

 

  List of verbs normally followed by Infi nitives

afford | agree | appear | arrange | ask | attempt | care | choose | claim | come | consent dare | decide | demand | deserve | determine | elect | endeavour | expect | fail | get | guarentee hate | help | hesitate | hope | hurry | incline | intend | learn | long | manage | mean | need offer | plan | prepare | pretend | promise | refuse | resolve | say | seem | tend | threaten | want | wish

List of verbs that can only have gerunds after t hem 

acknowledge admit | adore | anticipate | avoid | celebrate | confess | contemplate delay | deny || describe | detest | discuss || appreciate dislike | dread | endure | enjoy fancy | finish | imagine | involve | keep | justify | mention | mind | miss | omit | postpone | practise quit | recall | recommend | regret | report | resent | resume | risk | suggest | tolerate | understand e.g. anticipate implementing is correct (Anticipate to implement is wrong). Even though she didn't anticipate <to implement it, the advertising manager agreed with the personal> manager's proposal to strengthen her department. A) to implement it, the advertising manager agreed with the personal B) implementing it, the advertising manager agreed with the personal C) implementing it, the advertising manager agreed with the personnel D) to implement it, the advertising manager agreed with the personnel E) implementing it, the advertising manager agreed to the personnel (E) NOTE: all answer choices are indeed different here. 'Personal' deals with a person's own special things; 'Personnel' deals with a lot of people. Verbs like 'anticipate' must be followed by a gerund, the verb form in '-ing'. The correct idiom usage should read: a person 'agrees with' another person, not with an inanimate thing such as a proposal. Therefore, she agrees 'to the proposal.' (E) is correct.

10) THE EXPLETIVE ‘IT’ and 2 questions, testing the same fundamentals: 1) For many travellers, charter vacations often turn out to cost considerably more than they originally seemed. a. they originally seemed b. they originally seem to c. they seemingly would cost originally d. it seemed originally e. it originally seemed they would. B would need to be in past tense, seemed . The words original (and its derivations) and first usually require past tense. Furthermore, the "past future" of would in E is more precise than the simple past in A. Since we're talking about something we'd learn after a certain point in the past, would is better. For stores example: This website data such as This Acura is a lot better than I thought it would be. cookies to enable essential site is better than functionality,This as well Acuraas is marketing, a lot better than I thought it was. In the first example, weYou are saying that som something ething turned o out ut to b be e true. In the second example, we are saying that we personalization, and analytics. not aware of a fact that was true at that time. may change your settings at any time or accept And the in default settings. the charter vacations question, there is no fact that the travellers were unaware of when they purchased the charter

were

vacation--after the original purchase of the charter vacation (and probably toward the end of the vacation), the charter vacation turned out to be more costly than they had at first believed it would be. 2) Researchers are finding out that plastics are taking more time to deteriorate than they originally Privacy Policy

seemed.

A) They originally seemed.

Marketing B) they seemed originally

C) it seemed that they would originally

Personalization D) it originally seemed

E) it originally seemed they would

Analytics

'The tricky part is to choose between it and they. 'They' would imply that the plastics themselves first seemed to do one thing and then ended up doing another. Save Accept All The expletive 'it', the pronoun with no clear antedecent - makes more sense, it simply suggests that initial indications were misleading. Using 'it', it is necessary to include the phrase 'they would' to make it clear what seemed to be the case. So E is the best answer.

http://www.totalgadha.com

 

  It is as difficult to prevent crimes against property as those that are against a person. (A) those that are against a (B) those against a (C) it is against a (D) preventing those against a (E) it is to prevent those against a Answer is E, for the expletive it.

11) During  ‘during’ + time period is WRONG. WRONG. For example: During two hours, I felt sleepy. but During the last two hours, I have felt sleepy. For example: Even though its per capita food supply hardly increased during the two decades between 1940 and 1960...  To make our sentence correct with "during," we'd need to add some information that would identify which two decade-period we are talking about. Think of x as y ----- correct idiom (not ‘ to be’) 'So' is used to replace a Verb in a Sentence whereas  'It' 'So'  'It'  is used to replace a Noun. 

First, memorize the pattern: no sooner + inversion + than + sentence 

Quote: The answer was said to be D, but it seems that it should be E. The domesticated camel, which some scholars date around the twelfth century B.C., was w as the key to the development of the spice trade in the ancient world. a. The domesticated camel, which some scholars date b. The domesticated camel, which some scholars have thought to occur c. Domesticating the camel, dated by some scholars at d. The domestication of the camel, thought by some scholars to have occurred e. The camel's domestication, dated by some scholars to have been In D, it seems that "thought by some scholars..." modifies camel, rather than domestication E has a classic mistake, albeit a well disguised one! Example: The greatest change in my life was when I immigrated to t he US.

Can you see the mistake in this sentence? Another sentence, with a little hint: The greatest change in my life was when I immigrated to t he US.

Can you see the mistake now? Let us see one more sentence

This website stores data such as This pen is a bargain because it's only ten cents. cookies to again: enable essential site Hint This pen a bargain because it is only ten cents. functionality, asiswell as marketing, Okay, got it yet? personalization, and analytics. You may change your settingsThe at any time Let's work backward. last sentence is incorrect because it is incorrectly saying that the pen and a nd the ten cents are the same thing; a pen cannot be ten cents; it can be a writing instrument, it can be a bargain, it can even be a weapon in some cases, but it cannot be or accept the default settings. ten cents. One-tenth of a dollar is ten cents, a dime is ten cents, but a pen is not. Okay, now let's have a look at the immigration sentence: was when when  I immigrated to the US.

greatest change in my life PrivacyThe Policy

This sentence means that "change" and "when "w hen I immigrated..." are the same thing; they in fact are not. Marketing This is a classic mistake, and the classic correction is: The greatest change in my life occurred/happened when I immigrated to the US.

Personalization

(Do you see where I'm heading now???). So, in our original question, E says:

Analytics The camel's domestication was around the twelfth century B.C....

GMAT cleverly hides this mistake by using "to have been" instead of a simple be verb, but "to have been" is one of the many variants of was, were, is, isare, am, etc. The funny thing that GMAT uses the classic correction as well: Save Accept All domestication... occurred... when...

http://www.totalgadha.com

 

  NOUN + BE-VERB + NOUN/ADJECTIVE

For example: The change was good  good for me. The change was a good one  one for me. The change was an important step step  for me in my life. BUT NEVER The change was when I came to the US.

In other words, noun complements (the words that come after a be-verb and modify nouns) should only be nouns or adjectives (although we often use adverbs when we want to describe location).

Quote: 6. Why the answer is A? I picked E The central issue before the court was how far the regulatory agencies should go in requiring better working conditions in factories. a. in requiring better working conditions in factories b. as far as requiring better working conditions in factories c. in their requirement that factories should have better working conditions d. as far as requiring that factories should have better working conditions e. to require factories to hav e better working conditons Whew! What a tricky question! Both A and E are grammatically correct, but they have a very slight difference in meaning. For this question, we most likely want the meaning in A, not the meaning in E. And, whenever we have two options that are both grammatically correct, and the only difference is one of meaning, we MUST go with the original meaning. In other words, if A is grammatically correct, not wordy, redundant, awkward, etc., and another answer choice is also grammatically correct, not wordy, redundant, awkward, etc., we must go with A. But I'm sure you want to know the meaning difference and the rule, right? Okay, here you go: I know you won't like this, but with this meaning, we use "in." For example: I wantyou to know how from far you will gowe intalked helping me."helpful" + "in," right? Well, this is very similar--"helping me" is a process. In I think remember class that about

this sentence, I am wondering how long you would stay with me while you are helping me, how many different things you would do to help me. For example, would you break the law while you are helping me if you thought I would benefit? Would you ignore your friends and family while you are helping me? Again: During the process, how much would you do? (A) has a similar meaning in this sentence. Let's now look at the meaning of E. I want to know how far you will go to help me.

In this sentence, we are using the infinitive of purpose, which we use to express a goal. If I use this structure, I am wondering how much effort you would expend to help me. In other words, would you come to me at midnight? Would you travel 50 miles, 100 miles, 1,000 miles to help me? Would you spend all your time and money to come to help me? Would you give up your job, health, and family to help me? Again: How much would you do to be able to be in a situation to help me?? I know that these two are very, very close in meaning, but read what I've written very carefully, and be sure to post back with any further questions!

quote: 3. Why A is correct? I chose C Although about 99 percent of the more than 50 million Turks are Muslims, the republic founded by Mustafa Kemal Ataturk in 1923 is resolutely secular. This website stores data such as a... b. Although 99 percent cookies to enableabout essential site of over 50 million of the c. Although about 99 percent of more than 50 million functionality, as the wellfact as that marketing, d. Despite about 99 percent of more than 50 million personalization, andfact analytics. You e. Despite the that about 99 percent of over 50 million

may change your settings at any time Whew! This is one of the most commonly asked questions... I think it's going to take a while to explain, and I don't think I can do it or accept thesince default tonight I'vesettings. got class in the morning. Here's the short answer: if we use "the," we are saying that there are only 50 million Turks in the whole world; if we don't use "the," we are saying that there are possibly more than 50 million T urks in the world. PrivacyThis Policy one's similar to the one in the Official Guide, the one about the "Thomas Jefferson... setting free the more than 500 slaves..." ‘invest in’ is slightly preferable to ‘invest into’. into’.  There's also a very slight difference in meaning--"invest in" would be the better choice for such traditional investments as stocks and Marketing bonds, while "invest into" could be used in more metaphorical investments, such as the time, energy, and love you might shower upon your children. Personalization quote: Analytics The visiting doctors concluded that the present amalgam is probably as good as or better than, any other system that might be devised for the patients. This is correct. OneAccept of the answer Save All choices used 'might' instead of 'may'... what's the difference between m a y and m i g h t ?  In general, may has more of a concrete meaning, so should therefore be used more in statements of fact, whereas might is a bit less tangible, and tends to be used more in expressions of things that don't yet exist (hypothetical situations). Also, a bit more simply, since might is the past tense form of may , we use might more in the past tense.

http://www.totalgadha.com

 

  All that said, we often use them interchangeably in many constructions--there is a lot of overlap between may and might . STH (verb): give responsibility for. Thomas Edison is credited with inventing the light bulb. • credit SB with STH  • credit X to Y Y  (verb): give money or credit to. The bank credited $1 million to Trebla's account. • credit for  for (noun): money received for or in exchange for something. The customer received a $20 credit for the interruption in service.

So there are a few things you need to know here for GMAT Sentence Correction. First is this--you should know that GMAT likes to test you on "thinking words." T hese are words that indicate some sort of mental process, such as believe, belief, idea, theory, notion, concept , etc. Please note that both verbs and nouns can be considered "thinking words." GMAT typically likes to follow these words with that and a sentence. For example, on the GMAT it's better to say: • Lucise's belief that the Earth is flat was easily accepted. than to say  • Lucise's belief of the Earth being flat was easily accepted. It is okay to use ‘ of’ if we want to indicate only a noun. That's why, for example, we say theory of relativity . In this case, if we choose answer choices that use of instead of that , we seem to be talking more about theories of land mammals; we are not identifying the action of those land mammals. In other words, with the ‘ the  ‘that  mammals   that ’  ’,  we are leaving out what it is that the theory purports the mammals did. Crises is the plural of crisis Data is plural of datum

We generally use do to replace "regular" verbs, i.e., verbs that are not linking verbs, verbs that use modals, etc. For example:  M e g u m i s p e ak s J ap an es e be tt er th an I d o.   Look at the following examples for something (perhaps) new:  M e g u m i h as v i s i t e d m or e co u n t r i e s th an I h av e. We  M e gcan u m use i h ashave m oragain e s k i rbecause ts t h an have I d o. is an auxiliary verb here. Here, has is NOT an auxiliary verb, we cannot use the verb have in the second bit. Here's what you need to know: having + past participle is used to express actions that are finished finished  and to show that one thing comes after  after another. Furthermore, there is usually a "because relationship between the two. For example: Having eaten already, I turned down Megumi's invitation to dinner. This sentence is okay. But this next sentence is NOT okay, because the two things should be happening at the same time. Having been sick and having felt tired, Alan did not want to go to work. All the things in this sentence are happening at the same time, so we should NOT use the "having + past participle" construction here. And this sentence is incorrect because there's no "because relationship" between the two parts of the sentence: Having set, the Sun rose some hours later. The Sun will set and rise no matter what; w hat; setting doesn't cause rising, so we shouldn't use the "having + past participle" construction here.

This website stores data such as ‘Modeled After’  is the correct cookies to enable essential site idiom functionality, as welllm aso smarketing, H o p e f u ll ll y i s a lm t always w rong on GMA T personalization, and analytics. You use time Usage N o t e :settings  Writers who hopefully as a sentence adverb, as in Hopefully the measures will be adopted, should be aware that the may change your at any usage is unacceptable to many critics, including a large majority of the Usage Panel. It is not easy to explain why critics dislike this use or accept the default settings. of hopefully. The use is justified by analogy to similar uses of many other adverbs, as in Mercifully, the play was brief or Frankly, I have no use for your friend. And though this use of hopefully may have been a vogue word when it first gained currency back in the early 1960s, it has long since lost any hint of jargon or pretentiousness for the general reader. The wide acceptance of the usage reflects popular recognition of its usefulness; there is no precise substitute. Someone who says Hopefully, the treaty will be ratified Privacymakes Policy a hopeful prediction about the fate of the treaty, whereas someone who says I hope (or We hope or It is hoped) the treaty will be ratified expresses a bald statement about what is desired. Only the latter could be continued with a clause such as but it isn't likely. Marketing ·It might have been expected, then, that the initial flurry of objections to hopefully would have subsided once the usage became well established. Instead, critics appear to have become more adamant in their opposition. In the 1969 Usage Panel survey, 44 percent of the Panel Personalization approved the usage, but this dropped to 27 percent in our 1986 survey. (By contrast, 60 percent in the latter survey accepted the comparable Analytics use of mercifully in the sentence Mercifully, the game ended before the opponents could add another touchdown to the lopsided score.) It is not the use of sentence adverbs per se that bothers the Panel; rather, the specific use of hopefully in this way has become a shibboleth.

Save

Accept All

 J oj o h ad s o li tt le m on e y w h en s h e w as i n co lle g e th at s h e co u ld n ' t e v e n af f o r d to bu y n e w cl ot h e s , m u ch le s s ta k e a va c a t i o n . However much United States voters may agree that there is waste in government and that the government as a whole spends beyond

http://www.totalgadha.com

 

  its means, it is difficult to find broad support for a movement toward a minimal state. (A) However much United States voters may agree that (B) Despite the agreement among United States voters to the fact (C) Although United States voters agree (D) Even though United States voters may agree (E) There is agreement among United States voters that This is a very commonly asked question. The reason C is not the answer is that C changes the meaning. Look at these simplified sentences: • However much you complain, I will not change my m ind. This sentence means no matter how much you complain, I will not change my mind . Or, to put it in a more precise way, my resolve to stick to my decision will not wane even if the degree of your complaining increases. This meaning is quite specific. Now compare it to this sentence: • Although you complain, I will not change my mind. This sentence means even though you complain, I wil l not change my mind. So, even though the two meanings are quite close, they are in fact different, and between two grammatically correct and plausible sentences, we must go with the one that doesn't change the meaning of A. What does does ‘  ‘that which’ refer to in this sentence. (correct answer is D) The inhabitants of Somalia greeted the measures outlawing polygamy with a similar defiance that welcomed the prohibition of alcohol in the United States in the nineteen-twenties. a... b. a similar defiance which welcomed c. a similar defiance to what welcomed d. a defiance similar to that which welcomed e. the same defiance welcoming In English, instead of saying something like ‘that that’ we say ‘that which’.   For example: The number we recorded this week is greater than that which we w e recorded last week. is preferable to

The number we recorded this week is greater than that that we recorded last week. In this sentence The number we recorded this week is greater than that which we w e recorded last week. that = number and which = number Notice that this sentence equals The number we recorded this week is greater than the number that we recorded last week. So in our sentence here, we want to say: ance  that The inhabitants of Somalia greeted the measures outlawing polygamy with a defiance that was similar to the defi ance welcomed the prohibition of alcohol in the United States in the nineteen-twenties.

This website stores data such as This a greatessential SC trick! site cookies to isenable functionality, as well as marketing, Adjectives modify nouns; adverbs modify verbs, adjectives, and other adverbs.Sometimes in SC we must choose which to use personalization, according toand the analytics. meaning. You • supposed predecessors .  may change yourMediterranean settings at any time This sentence means that we are not sure whether these things are actually predecessors.  or accept the default settings. • supposedly Mediterranean Mediterranean  predecessors. This sentence means that we are not sure whether these things are actually Mediterranean. 

PrivacyShort Policy and Sweet Sweet   Marketing GMAC prefers concise or shorter formulations over longer, wordier ones The GMAT prefers conciseness over wordiness. If you are left with two answer choices that appear to be grammatically correct, select the shorter and more succinct option. Personalization Short and sweet comes into play only after you have corrected any grammatical errors in the original sentence. Analytics Which answer choices correct the error in this sentence? In this example, (A) and (D) are incorrect because of an Apples and Oranges problem. The ’distribution of Accept mass’ cannot Save All be compared directly to the ’mantle.’ Answers (B), (C), and (E) all attempt to correct the problem and compare the ’distribution of mass’ to ’distribution of mass,’ either directly or by using the pronoun ’that.’ Can you choose among (B), (C), and (E)?

http://www.totalgadha.com

 

  Although it’s close, we can eliminate answer choice (C) because it refers to the distribution of mass of the mantle. We want to compare distribution of mass within the core and within the mantle. Between (B) and (E), we can apply the short and sweet technique. Although (E) is grammatically correct, answer choice (C) expresses the same idea more concisely and is the correct answer. Redundancy

Sentences that include repetitive or redundant words or phrases should be avoided. Sometimes authors repeat themselves within a sentence. GMAC considers this an error. Some typical redundancy examples: • regain… again… • rise ...up ... • decline ...down ... • It is likely that ...may . • soar ...up ... • decrease ...down • re-Verb ...again ... • the reason ... is because • the reason why .. is because • close… proximity • true… fact • circulate… around • Attempt…try • Can…potential • Although…but Can you find the redundancy in this sentence? ’Plummeting values . . . have fallen’ is redundant and therefore wrong. Find the answer Choice that eliminates the redundancy. Answer choice (C) corrects the error by eliminating the second reference to a decrease and is therefore the best answer .

This website stores data such as cookies to enable essential site functionality, as well as marketing, personalization, and analytics. You may change your settings at any time or accept the default settings.

Privacy Policy Marketing Personalization Analytics Save

Accept All

http://www.totalgadha.com

 

  Reading Comprehension Study Strategy

General Strategies for Reading Comprehension, by Stephen Bolton

1.  Try to read the whole text of t he passage once, if possible. Many people think you should just skim the passage or read 1. the first lines of every paragraph, and not to read the passage. We believe this is an error: if you misunderstand the main idea of the passage, you will certainly get at least some of the questions wrong. Give the passage one good read, taking no more than 3 minutes to read all of the text. Do not read the passage more than once – that wastes too much time. If you have not understood it completely, try to answer the questions anyway. Note: this point of reading the whole passage is important for test-takers whose first language is not English, provided that they can read the passage in 3 minutes or less.

2. Make brief notes on the text on your scrap paper.  As we will see below in greater detail, you should write down a couple of words on A) the Main Idea or Primary Purpose, B) Organization/Structure of the passage, and C) the Tone or Attitude of the author (if applicable). You just need a few words for each of these areas, and altogether it should not take longer than 30 seconds to write down.

3. Remember that the tone or attitude of the passage is usually respectful and moderate, never going to extremes of praise nor criticism. ETS obtains its Reading Comprehension passages from real articles about real academics and

professionals. So the tone of the articles, even when there is criticism in the passage toward an academic a cademic or her work, is always balanced and moderate. In the same vein, articles that deal dea l with minorities or ethnic groups are almost always positive and sympathetic.

4. Look out for structural words that t ell you the important ideas or transitions in a passage.  Continue the Idea Words 

Similarly Moreover Additionally In the same way Likewise Conclusion Words 

Thus Therefore Hence So In summary stores data such In conclusion

This website as cookies to enable essential site Contradiction or Contrast Words   functionality, as well as marketing, Neverthless personalization, and analytics. You Nonetheless However may change your settings at any time But or accept the default settings. Although Though Even though Notwithstanding Privacy Policy Yet Despite Marketing In spite of On the one hand…on the other hand While Personalization Unlike

Analytics Save

5. Go back to the text of the passage for the answers. Many test-takers fail to return to the text of the passage to look for the correct answers. They rely solely on their memories and understanding of the passage after having read or skimmed it. Wrong. ETS is counting Accept All on that. Go back to the text to look for information to answer the questions. Nine times out of ten, the answer lies within the passage.

Of the 6 most important types of questions for Reading Comprehension, we will first look at Main Idea/Primary

http://www.totalgadha.com

 

  Purpose Questions, and the strategies we can use to answer them. Main Idea/Primary Purpose Questions

Many people believe there is no difference between the main or central idea of the passage and the primary purpose of the author of the passage. This is simply not true. Let's take a look at the subtle but important difference between them: Main Idea 

The question might look something like this: ’ Which of the following best states the central idea of the passage?’ ’Which of the following most accurately states the main idea of the passage? ’Which of the following is the principal topic of the passage?’ ’The main topic of the passage is....’ Primary Purpose 

The question might look like this: "The primary purpose of this passage is to..." "The primary purpose of the passage as a whole is to..." "The primary focus of this passage is on which of the following?" "The main concern of the passage is to..." "In the passage, the author is primarily interested in...." "The passage is chiefly concerned with..." Strategy:  Main Idea: Look in the first and last paragraphs for the main idea. Any conclusion words like therefore, thus, so, hence, etc. that you see are most likely introducing the main idea. The correct answer will say the same

thing as it says in the text, but using different words. The Main Idea is not always stated explicitly in the passage – in fact, more likely than not, it is not stated explicitly. Therefore, in order to answer this type of question when it is more implicit: 1. Re-read the first line of every passage, passage, and the last line of the first and last paragraphs. This should give you general structure outlinestructure of the argument, with which you can eliminate answer the Main Idea question. 2. the After determining theor general or content of the argument, answer choices that are too broad or too specific, i.e. answer choices that go beyond the content of the passage, or that deal with content only discussed in one paragraph of the passage. 3. Make brief notes – a couple of words- regarding the Main Idea on the text on your scrap paper while you read. Primary Purpose: What is the author trying to do? What is his intention? If he is evaluating a theory, then the answer could be something like "Discuss an interpretation". Note that the correct answer would deal with " an interpretation", because the author is only dealing with one theory. If the Primary Purpose is to criticize 2 new books, then his intention or his primary purpose might be to "Critique new studies". Again, as in Main Idea questions, re-read the first line of every passage, and the last line of the first  and  and last  paragraphs.  paragraphs. This should give you the general structure or outline of the argument, with which you can answer the Primary Purpose question. Note: A good main idea or primary purpose does not go beyond the scope of the passage, nor does it limit itself to discussing only one part of the passage.

What is the primary purpose of this passage? A) discuss the importance of the television program Star Trek for the international space program B) discuss important theoretical work concerned with faster-than-light space travel. C)dataexplore a dispute among theoretical physicists regarding the uses of space flight stores such as

This website describe the possible uses of space-warping material cookies to enable D) essential sitehow E) explain a space-warping bubble would work in the real world functionality, as well as marketing, Explanation   analytics. You personalization, and This is a Primary Purpose question, so we have to determine what the author is trying to do or say in this passage. So, let's read the may change your settings at any time first and last lines of the passage in order to get an idea of the primary purpose. The first line says "Great news for Star Trek fans: or accept default warpthe drives that settings. can propel starships around the Galaxy faster than the speed of light may be possible after all--with all--w ith a little help from Dr Who." The last line is a quote by a physicist that says "Of course, there are still some basic questions--like how does one go about constructing this Tardis space-time--but it puts the concept of space warps back on the agenda." From both these sentences, we get the idea of space travel, faster than light travel and space warps – maybe this is a discussion of faster than light space travel. Does Privacythat Policy match what you have already read? Yes, basically this is a discussion of the theoretical state of play in the area of faster-than-light space travel. Do any of the 5 answer choices match that? Yes – B, even if the wording is somewhat different from how we are wording it, the idea is almost exactly the same. B is the answer. Marketing Another way of getting to the answer is through elimination of obviously incorrect answer choices. We can eliminate A because the author mentions the popular science fiction program Star Trek merely to introduce the idea of faster-than-light travel, and nothing Personalization more. C is a stronger possibility because the second paragraph of the passage does discuss some disagreement among physicists about the possibility of creating a warp-drive, but in the same paragraph the theoretical dilemma seems resolved. Moreover, since the Analytics author only discusses this in one paragraph, it cannot be the primary purpose of the entire passage. We can e liminate D because the author does not go into detail discussing the uses of space-warping material. And we can discard E because the author does not really go into how the space-warping Save Accept Allbubble would work in the real world. Title Questions 

Title questions are very similar to Main Idea questions, though are less common. Though some of the example passage we use in this

http://www.totalgadha.com

 

  tutorial and in the Practice Section are from the New Scientist, and therefore have titles, the passages in the real GMAT will not have titles. The question might look like this: "Which of the following titles best summarizes the passage as a whole?" Strategy

Treat this as a Main Idea question. A good title sums up the central idea of a passage. Therefore, in order to answer this type of question: 1. Look in the first and last paragraphs for the main idea. Any conclusion words like therefore, thus, so, hence, e tc. that you see are most likely introducing the Main Idea/Title. The correct answer will say the same thing as it says in the text, but using different words. 2. Re-read the first line of every passage, and the last line of the first and last paragraphs. This should give you the general structure or outline of the argument, with which you can answer the Title question. 3. Make brief notes – a couple of words- regarding the Title on the text on your scrap paper while you read. 4. After determining the general structure or content of the argument, eliminate answer choices that are too broad or too specific, i.e. answer choices that go beyond the content of the passage, or that deal with content only discussed in one paragraph of the passage.

What would be an appropriate title for this passage? A)

Constructing The Tardis

B)

How To Make Space-Warping Material

C)

Bubbles In Space-Time

D)

Faster-Than-Light Travel: A Possibility?

E)

Debate On The Uses of Space Travel

Explanation 

This passage actually already has a title, "Warp Factor One". But we have to look for another title possibility, one that would be most like the Main Idea of the passage. We look at the first and last la st paragraphs, and since the Main Idea is that researchers now feel that faster-than-light travel maybe more than mere fantasy, we can find the correct answer choice. Does any answer choice correspond to this idea? Yes- answer D, which is the correct answer. We can also find the correct answer through elimination. There is nowhere in the passage where it discusses building Dr. Who's Tardis (pity!), so we can eliminate A. Nor does it tell us how to make space-warping material. Eliminate B. While bubbles in space-time are discussed at some length in one of the paragraphs, we cannot say this is the main concern of the passage, and thus should eliminate C. And nowhere are the uses of space travel discussed, so discard E. Specific Detail or Target questions are probably the most common types of questions, and the easiest to answer. The question might look like this: "According to the passage,...."

This website stores data such as...." "The passage states that cookies to enable essential site Strategy functionality, as  well as marketing, The Specificand Detail or TargetYou that we are looking for could be a Line Number, or a Name or Date. Go to the Line Number or Name or personalization, analytics. Date, and then read several lines above and below it. Find the answer choice that basically says the same thing as in the passage, may change your settings at any time or accept theusually defaultwith settings. though different words or word order. According to the passage, Pfenning and Ford A)

demonstrated conclusively the impossibility of faster-than-light travel

B)

explored the possibility of bubbles that warp space

Privacy Policy

C) supported the work of Alcubierre Marketing D)

work at of the the Institute for Theoretical Physics at the Catholic University of Leuven

E)

suggested that a warp drive was not physically possible

Personalization Analytics Explanation 

Save Accept Allquestion, and therefore we look for the Name, Line Number, or Date that will help us. In this case, the This is a Specific Detail/Target detail consists of the names Pfenning and Ford. We scan the text, starting from the top of the passage, looking for the names Pfenning

http://www.totalgadha.com

 

  and Ford. We find them in only place, at the beginning of the second paragraph. We read a couple of lines above the names, and keep reading until a few lines after the names. It says "But in 1997 Michael Pfenning and Larry Ford at Tufts University in Medford, Massachusetts, apparently killed this ingenious idea by showing that it needed far more than the entire energy content of the Universe to work (This Week, 26 July 1997, p 6)". The line after that says the research of another physicist then resurrected the possibility of FTL travel, negating the implications of the research of Pfennig and Ford. Now we can answer the question. Do a any ny of the answer choices match the information given around the target area? Yes- E. Let's also eliminate. If we re-read what th e passage says about Pfenning and Ford, we can eliminate B, C, and D. None of them are supported by the information in the passage, so let's eliminate all of them without wasting too much time and with a minimum of fuss. A is tougher to eliminate. From the sentence that mention Pfenning and Ford, it seems their work does rule out the possibility of a space-warp drive. But if we read the next line, it says another researcher said it was indeed possible. So the Pfenning and Ford could not have "conclusively" demonstrated the impossibity of the FTL drive.

This is probably the most difficult type of Reading Comprehension problem. The question might look like this: "It can be inferred that the author makes which of the following assumptions?" "Which is an assumption underlying the last sentence of the passage?" "Which of the following, if true, would most strengthen the hypothesis mentioned in lines 17-19?" "With which of the following statements regarding chaos theory would the author be most likely to agree?" Strategy:

1. First, treat this type of problem as a Specific Target question. Look for a target in the question, find it in the text, and then look above and below it. Often you do not have to infer  very  very much, the answer remains within the text. 2. If the answer must be inferred and is not stated explicitly within the text, then choose the answer choice that can be inferred or assumed from the information given. Again, you should not have to infer very much – only one or two logical steps removed from the information in the passage. 3. Make sure that the answer choice you decide on does not violate or contradict the Main Ma in Idea of the passage - if it does, the answer choice is probably wrong. It can A) B) C) D) E)

be inferred that a house with the propeties of the bubble mentioned in the passage would be larger on the inside than on the outside could move faster than the speed of light might be very energy efficient could move through time would eventually fold in on itself and be destroyed

Explanation: 

First, let's try to deal with this question as a Specific Target problem. Is there a target in the question? Yes – the bubble. The bubble is first mentioned at the end of the second paragraph, and then discussed at length throughout the third paragraph. Remember, we have to look above and below that target area (as well as read the target area again), so quickly go through the second, third, and first part of the fourth paragraph. When you are finished, look at the answer choices. Can any of them be inferred from the information given in the target area? Well, we could eliminate C, D, and E for simply not being supported by the information given in the passage. B – maybe, but a This website stores data such as house moving through time seems pretty silly. But in the fourth paragraph the author talks about the Tardis, "wh ich looked like a boxessential but had asite spacious interior". Big on the inside, small on the outside. Is that like our house? Yes- answer A. As well, we cookies to police enable can choose A because it does not go against or contradict the Main Idea in this case, which if it had, would have made it ne cessary functionality, as well as to eliminate. Somarketing, choose A.

personalization, and analytics. You Theyour question might this: may change settings atlook anylike time "The author's attitude towards Morgan's theory could best be described as one of ..." or accept the default settings. Strategy :

Look for descriptive words, adjectives or adverbs, that could tell you the author's attitude. For example, the words unfortunately  or  suggest a negative connotation, while strength or valuable emphasize the positive. Make brief notes – a couple of wordsflaw  suggest regarding the Tone of the text on your scrap paper while you read. Additionally, keep in mind that the author's attitude toward a Privacytheory, Policybook, or ethnic group will almost always be respectful, even when somewhat critical.

MarketingThe author's attitude towards Miguel Alcibierre's theory could best be described as one of A)

Admiration

C) D) E)

unbridled scorn Dismay complete objectivity

B) mild scepticism Personalization

Analytics

Save Explanation:  

Accept All

Since this is a Tone/Attitude question, we must look in the passage for descriptive words that tell us what the author thinks of Alcibierre and his theory. In the second paragraph the author call's Alcibierre's theory "this ingenious idea". This is positive, and the only positive answer choice is A. A is the correct answer. As well, if we could not find the tone so easily, we could also eliminate C and

http://www.totalgadha.com

 

  D at the very least, for being too extreme. The question might look like this: "Which of the following best describes the organization of the passage?" "Which of the following best describes the organization of the first paragraph of the passage?" "One function of the third paragraph is to...." Strategy: Re-read the first line of every passage, and the last line of the first  and  and last  paragraphs.  paragraphs. This should give you the general structure or outline of the argument, with which you can answer the question. Remember to make brief notes about the structure of the text on your scrap paper. If you are looking for the organization of one paragraph, read the first and second sentence of the paragraph. That will give you a rough idea of what is the structure or organization of the paragraph. Which of the following best describes the organization of the second paragraph of the passage? Two investigations that support Alcubierre's theory are introduced A) B) Possible objections to to the uses of the warp drive are present, and then refuted refuted An objection to the practicality of the theory is raised, and then another work is cited to shore up the C) applicability of the original theory A work of theoretical physics that supports Alcubierre's theory is raised, and then another that refutes it is D) presented E) Alcubierre's theory is analyzed by a panel of several eminent physicists Explanation: 

Read the first sentence of the paragraph: "But in 1997 Michael Pfenning and Larry Ford at Tufts University in Medford, Massachusetts, apparently killed this ingenious idea by showing that it needed far more than the entire energy content of the Universe to work (This Week, 26 July 1997, p 6)". Then read the second sentence: "Now Chris Van Den Broeck of the Institute for Theoretical Physics at the Catholic University of Leuven, Belgium, has resurrected Alcubierre's proposal". So if we out those two sentences together, and in different words, first the usefulness of Alcubierre's theory is questioned by two researchers, then the theory is validated by yet another researcher. Which of the answer choices is closest to this? C. None of the other answer choices follow the organizational pattern of the paragraph – they reverse it, or are completely dissimilar. C is the only possible answer. 1. Read the whole text of the passage once. once. 2. Make brief notes ab about out the text on your scrap paper. 3. Remember that the tone or attitude of the passage is usually respectful and moderate, never g going oing to extremes of praise nor criticism. 4. Look out for structural words that tell you the important ideas or transitions in a passage. 5. Go back to the text of the p passage assage for the answers to specific questions. questions.

Test Taking Strategy 1)  Aggressively read each paragraph for its main idea. If you can’t write down in a few words what the p point oint of each paragraph

is, you weren’t reading actively enough. You should jot down the following. •  Main idea or primary purpose •  Organization/Structure •  Tone or attitude of author (if applicable) Note: Be careful to not write facts down. It’ll bog you down and usually results in a loss of of the big picture and moves

you to focus too much into the details. 2)  stores Note any trigger This website data suchwords, as same train of thought words, yin-yang parallelism.   Weed cookies to 3) enable essential sitedisputable answers. Vague, wimpy answers are often correct over out possible possible over stronger statements. ETS doesn’t want to get many complaints that a particular answer that was strongly strongly stated, that exceptions could arise. ETS would rather play functionality, as well as marketing, it safe. 4)  Minority passages are tone and answers tend to be positive in tone as well. Again, ETS, wouldn’t want to look personalization, and analytics. Youoften positive in tone prejudiced. may change your settings at any time 5)  Always eliminate bad choices first before answering. You’ll almost always be able to narrow down to 2-3 and that significantly or accept the default settings. improves your odds of getting the question right. 6)  Read the entire passage before answering the questions. Other books say skim, but it’s not always successful with more difficult passages where minute details change meanings of the passage and could get you going down the wrong path on inference or main point point questions. Give yourself 3 minutes or less. Privacy Policy 7)  For Inference questions, (Note: these are usually the hardest of all RC questions) go find the general area being referenced. Read a bit above or below below it and then make your choices. Don’t go by memory. This is going to cause more problems than than are helpful with saving time in the long run. Your answer should never contradict the main p point oint of the passage. Marketing 8)  Most people get main point and inference questions wrong so focus more carefully on these. 9)  "According to the passage/author" question type type of questions. Whenever you see this question, tell yourself, "Stop and stop Personalization thinking. I need to FIND, not think." 10)  For main point or central idea type of questions, re-read the first and last sentences of each paragraph before making Analytics elimination choices and answering. Getting the overall structure structure is really helpful before answering. 11)  In Summary: Consider weeding out answers that

Save •  • 

• 

AccepttoAll Are disrespectful others/professionals. ETS doesn’t like to be disrespectf others/professionals. disrespectful. ul. Too strong an answer. Use of words like “only”, “definitely”, “positively” Condone/approve prejudicial attitudes. ETS doesn’t like to be disrespectful.

http://www.totalgadha.com

 

 

This website stores data such as cookies to enable essential site functionality, as well as marketing, personalization, and analytics. You may change your settings at any time or accept the default settings.

Privacy Policy Marketing Personalization Analytics Save

Accept All

Critical Reasoning Strategy

[We want to warn you immediately that this strategy is not the easiest way to do CR (the easiest would be read-and-answer), but it lets you get the most questions right spending less time per correct answer. The bottom line is that it won't be easy to follow this

http://www.totalgadha.com

 

  strategy but if you do, it will reward you]

1.  Read the question (this needed so that you would know what to look for and what to do: find an assumption, weaken, infer something or else; do not worry about the details in the question, read for keywords, such as strengthen, deny, or explain. Sometimes at the end of the Verbal section on the test, your brain won't even hol d a keyword, so you may want to write it down symbolically; e.g. + for strengthen or - for weaken. 2.  Read the passage (Read it very attentively because in contrast to Reading Comprehension, there is very little text here and mostly everything is important; try to read only once. Reread only hard texts). 3.  As you read, look for the problem in the passage (evaluate how convincing it is) 4.  Paraphrase the passage (this a very important step because when you do a paraphrase, you check whether you understood the passage and at the same time you extract the skeleton of the argument, making it easier to identify the conclusion and the assumption. Very often, the paraphrase of the passage will be pretty close to the conclusion. It is not surprising, since the

5.  6. 

7.  8. 

conclusion is the main point and evidence just supports it.) Your paraphrase should be as close to the text and as simple as possible so that you would understand it easily and at the same time could fully trust it. Do not make it too general nor too detail oriented. When you do a paraphrase, do it in three steps: Evidence1, Evidence2, and Conclusion; put "therefore" word before you start your conclusion, this will help you to set it off. Read the question again (now with more understanding of what is being asked; reading the question 2 times, will also help you to make sure you answer exactly what is stated and that you understand the question.) Answer before reading the answer choices (Why do this? Two reasons: one, if you can think of the correct answer or at least the general direction that the answer choice needs to be, you will identify it among the wrong choices much faster, thus spend less time reading the answers, which usually take 30 seconds to cover. The second reason is that often test takers are seduced by the author's wording. One reads a few words that were used in the passage and the brain identifies this choice with the passage, thus making it seem more right that it needs to be. The more problems you practice with, the more chance is you will guess the right answer even before reading it. And there is nothing more pleasant than seeing YOUR answer choices among the listed. ) Go through the answers, first time scan them for YOUR answer choice (usually you will guess correctly in 60-70% of cases), if you did not find it, reread them more attentively. Draw a grid to eliminate the wrong answers easier. Use "+" "+ " for a sure answer, "-" for a definitely wrong answer choice, and "~" or "?" for an answer that may be right or questionable. This will help to concentrate only on a few answer choices and will prevent you from reading same answers several times if you get confused or keep having troubles locating the right answer. A +

B ? C D E + 9.  Always remember to think how the answer choice relates EXACTLY to this situation; it may be out of scope by being too general. E.g. Advertisement: For sinus pain, three out of four hospitals give their patients Novex. So when you want the most effective painkille r for sinus pain, Novex is the one to choose. Which of the following, if true, most seriously undermines the advertiseme nt's argument? (A) -(B) -(C) Many drug manufacturers increase sales of their products to hospitals by sell ing these products to the hospitals at the lowest price the manufacturers ca n afford. (D) Unlike some competing brands of painkillers, Novex is available from pha rmacies without a doctor's prescription. ( E ) - -stores data such as website

This cookies to enable essential site As about this session we will be mostly concerned with one of the most important parts of a GMAT argument, Assumption. Many CR functionality, asdirectly well asask marketing, questions for an assumption or are based on them, such as weaken and strengthen questions. Also, assumption of an argument is one of the only You parts that we can influence to destroy or solidify an argument; thus if we can disprove an assumption, we personalization, and analytics. can negate the whole argument because the conclusion will not make sense. On the other hand, if we can strengthen our assumption, may change your settings at any time thus patch a possible hole, we will create a stronger argument and our conclusion will be more credible.  (Sometimes some of the or accept the default settings. evidence is doubted or is amended to add new meaning and change the argument, but generally, it is the assumption that is attacked to destroy an argument. In a case when evidence is completed with more info, we can still say that it is the assumption that the evidence we had was valid is being destroyed). Thus, to succeed in CR, it is crucial that you are able to extract an assumption fast; it will save you time on many questions. Practicing with assumptions does not take too long; 2-4 hours and your skills will be on top. PrivacyAlso, Policy you will be a better speaker and a more critical writer if you are able to see assumptions of the writer who argues against you because as we have said, if you can kill an assumption, you will damage the argument. For example, if we play with We will also cover one of the most important steps of the CR approach, paraphrase. Paraphrase allows you to check how well you understood the passage Marketing as well as to see the structure stripped. Paraphrase should not take you more than 5-10 seconds after you have read the passage. Sometimes it is useful to write down some complicated relationship or something that will help you to understand the passage better, Personalization such as a diagram, for example. Other than that, don't write down anything else, the passage that you read will be so short that you should be able to remember all the details without writing down anything. Analytics Today we were supposed to learn that the assumption  assumption of an argument is •  a bridge (link) between the evidence and the conclusion of an argument

Save •  •  • 

Accept never stated in the All text is the most vulnerable element of an argument is the only element in the argument that can be influenced

http://www.totalgadha.com

 

  • 

should be easy for you to find by now

Paraphrase  • 

Should be brief and take 5-10 seconds Will state the main idea of the passage and be close to the conclusion Will help you to understand the passage better Will reveal the conclusion, evidence, and eventually, the assumption Start your conclusion with Therefore

•  •  •  • 

Question # 

Paraphrase

Old study – eating chocolate increases chances of getting heart disease; new study – chocolate does not increase heart disease chances; therefore, people will buy more chocolate.

People were not buying chocolate because they were afraid of heart disease.

2

As climate of North America got hot and dry large mammals died while small ones survived.

3

Higher profits will give higher bonuses, therefore, general economic recession year will bring lower bonuses than year of profits.

This assumption is possible only after brining in an implied conclusion – change in climate killed the large animals. Large mammals were more sensitive to heat than small ones. The Industry will not bring profits in the year of general economic recession.

4

There’s less suitable area for gray wolves, however, bringing gray wolves back to the places where they have been hunted out is immoral.

People will keep hunting.

5

Developed a safer equipment for Swedish market, brought it to US but doesn’t advertise this safety improvement.

Safety improvement should be advertise because it will help him win over it’s competitors.

6

Few residents use public buses because there are enough automobiles, therefore, public bus system should not be subsidized.

Killing subsidy for the public bus system will not hurt the suburbs community.

7

A survey was conducted, according to is employees with high ratings ware satisfied with the company’s system, therefore, the company’s best performing employees like the system.

Employees with high ratings are the company’s best performing employees.

8

1970’s – fall in the average annual income of college graduates as compared to h igh school graduates;1980’s – the average annual income of college graduates increases while the number of data such as college graduates did not decrease.

This website stores cookies to enable essential site functionality, 9as well asIndustrialists marketing, are accused of intervention in war to make profit, however, federal expenses for personalization, and analytics. You intervention were larger than profits, therefore, may change your settings at any time the accusation is wrongly motivated. 10. School board decided to reduce its staff by laying or accept the default settings. off the least effective teachers first

11.

Privacy Policy

Marketing

Applied scientific research is emphasized because it leads to technological advancement; basic research should be paid attention to because it’s a base for applied research

12. Decision to invest in electronic system has cost Personalization advantage over nonelectronic system, therefore, it

Analytics 13.

Save

Assumption

1

will give advantage over competitors One museum sold 30K tickets, in a year 2 more museums opened and together the 3 of them sold Accept Alltherefore, museums were worth the 80K tickets, cost

N/A

Federal money (expenses and profits) and industrialists’ money come from the same source. It is possible to determine the level of teachers’ effectiveness Basic research does not lead to technological advancement

Competitors are not using the electronic system yet and won’t match it The first old museum did not sell more tickets next year, as compared to previous year, and it is due to the two new museums that the number of tickets sold increased

http://www.totalgadha.com

 

  14.

Company needs to cut costs, so offer early retirement first, and then fire others to have the overall reduction of costs to 50 percent

15.

Need to reduce airport congestion, so send passengers by rapid trains to several cities while realizing that it is the major airport that is congested

16.

Received 2000 letters, most of them support him, therefore, most people in the country support him

People who wrote the letters represent the opinion of majority

17.

Government wanted to regulate what can be shown in tobacco and alcohol advertisement, as a result, these ads became more inventive and humorous

Implied conclusion: tobacco and alcohol advertisement should be banned. 1. Use of tobacco and alcohol can be influenced by advertisements 2. The funny and inventive commercials are more persuasive than the old ones.

18.

Lead contamination dropped: federal regulation went into effect, but mainly because there was a drop in the use of leaded gasoline

N/A – no conclusion

19.

Decrease in traditional child disease, and at the same time increase in rare infection among children whereas few adults are affected

Children are more sensitive to rare infections than adults

20.

Plants come from less developed nations without compensation, but coal, oil and ores are extracted for payment

The value and benefit from these plants are the same as from coal, oil, etc. They are comparable

21.

Donates an exhibit which demands a storage space, a routine conservation, therefore, it adds to museum’s expenses and does not help it financially

The number of visitors to see the exhibit will not override the financial costs of keeping the exhibit

22.

Oil prices are remaining low, therefore, natural gas prices will also be low

Oil and natural gas prices are interconnected

Sources:Total Gadha Verbal Lessons Score Top GMAT Study Strategy Official Guide to GMAT GMAT Verbal Notes by Spidey GMAT Verbal Notes by Sumit This website stores data such as Amateurs GMAT Notes 2006 cookies to enable site GMAT Flash essential Cards GMAT% 20 Fact functionality, as well as marketing,

personalization, and analytics. You may change your Compiled by :settings at any time Dagny Taggart or accept the default settings.

Privacy Policy Marketing Personalization Analytics Save

Accept All

Reducing costs by firing will not hurt company’s productivity: 50 percent of managers left will be able to do the double work load Passengers will be willing to switch from air travel to trains

Sponsor Documents

Or use your account on DocShare.tips

Hide

Forgot your password?

Or register your new account on DocShare.tips

Hide

Lost your password? Please enter your email address. You will receive a link to create a new password.

Back to log-in

Close